NU270 Week 4 PrepU: Clinical Decision Making / Clinical Judgment

Pataasin ang iyong marka sa homework at exams ngayon gamit ang Quizwiz!

Which type of fracture involves a break through only part of the cross-section of the bone? Open Comminuted Oblique Incomplete

Incomplete Explanation: An incomplete fracture involves a break through only part of the cross-section of the bone. A comminuted fracture is one that produces several bone fragments. An open fracture is one in which the skin or mucous membrane wound extends to the fractured bone. An oblique fracture runs across the bone at a diagonal angle of 45 to 60 degrees. Reference: Hinkle, J.L., and Cheever, K.H. Brunner & Suddarth's Textbook of Medical-Surgical Nursing, 14th ed., Philadelphia: Wolters Kluwer, 2018, Chapter 42: Management of Patients With Musculoskeletal Trauma, Fractures, p. 1189. Chapter 42: Management of Patients With Musculoskeletal Trauma - Page 1189

The nurse is reviewing client lab work for a critical lab value. Which value is called to the physician for additional orders? Magnesium:2 mEq/L Calcium: 10 mg/dL Potassium: 5.8 mEq/L Sodium: 138 mEq/L

Potassium: 5.8 mEq/L Explanation: Normal potassium level is approximately 3.5 to 5.0 mEq/L. Elevated potassium levels can lead to muscle weakness, paresthesias, and cardiac dysrhythmias. Reference: Hinkle, J.L., and Cheever, K.H. Brunner & Suddarth's Textbook of Medical-Surgical Nursing, 14th ed., Philadelphia: Wolters Kluwer, 2018, Chapter 13: Fluid and Electrolytes: Balance and Disturbance, Potassium Excess (Hyperkalemia), p. 271. Chapter 13: Fluid and Electrolytes: Balance and Disturbance - Page 271

A client in an acute care mental health program refuses a morning dose of an oral antipsychotic medication and believes it contains poison. The nurse should respond by taking which action? consulting with the physician about a care plan. omitting the dose and trying again the next day administering the medication by injection crushing the medication and putting it in the client's food

consulting with the physician about a care plan. Explanation: To determine a care plan for clients who are noncompliant with medications, the nurse should consult with the physician. Unless there is a danger to self or others, the client can't be forced to take medications. Crushing the medication and putting it in food might make the client suspicious. The nurse shouldn't omit the dose and try again the next day. The nurse should instead make another attempt to administer the drug to avoid decreased drug levels.

A client has burns to his anterior trunk and left arm. Using the Rule of the Nines, what is the TBSA burned? 36% 27% 45% 18%

27% Explanation: The TBSA would be 27%. That is 18% of the body surface for the anterior trunk and 9% for the left arm. Reference: Hinkle, J.L., and Cheever, K.H. Brunner & Suddarth's Textbook of Medical-Surgical Nursing, 14th ed., Philadelphia: Wolters Kluwer, 2018, Chapter 62: Management of Patients with Burn Injury, Rule of Nines, p. 1848. Chapter 62: Management of Patients with Burn Injury - Page 1848

The nurse is auscultating the heart sounds of a patient with mitral stenosis. The pulse rhythm is weak and irregular. What rhythm does the nurse expect to see on the electrocardiogram (ECG)? Atrial fibrillation First-degree atrioventricular block Ventricular tachycardia Sinus dysrhythmia

Atrial fibrillation Explanation: In mitral stenosis, the pulse is weak and often irregular because of atrial fibrillation (caused by strain on the atrium). Reference: Hinkle, J.L., and Cheever, K.H. Brunner & Suddarth's Textbook of Medical-Surgical Nursing, 14th ed., Philadelphia: Wolters Kluwer, 2018, Chapter 28: Managements of Patients With Structural, Infectious and Inflammatory Cardiac Disorders, Assessment and Diagnostic Findings, p. 794. Chapter 28: Managements of Patients With Structural, Infectious and Inflammatory Cardiac Disorders - Page 794

Which value does the nurse recognize as the best clinical measure of renal function? Creatinine clearance Urine-specific gravity Volume of urine output Circulating ADH concentration

Creatinine clearance Explanation: Creatinine clearance is a good measure of the glomerular filtration rate (GFR), the amount of plasma filtered through the glomeruli per unit of time. Creatinine clearance is the best approximation of renal function. As renal function declines, both creatinine clearance and renal clearance (the ability to excrete solutes) decrease. Reference: Hinkle, J.L., and Cheever, K.H. Brunner & Suddarth's Textbook of Medical-Surgical Nursing, 14th ed., Philadelphia: Wolters Kluwer, 2018, Chapter 53: Assessment of Kidney and Urinary Function, p. 1553. Chapter 53: Assessment of Kidney and Urinary Function - Page 1553

A client who takes zinc daily is diagnosed with a severe infection and is ordered levofloxacin (Levaquin). The nurse is aware that taking these two drugs may have what affect on the antibiotic? Decreased elimination Decreased absorption Increased absorption Increased elimination

Decreased absorption Explanation: Antacids, iron salts and zinc can cause decreased absorption of the antibiotic. There is no affect on elimination of the antibiotic. Reference: Chapter 9: Antibiotics - Page 112

A client is prescribed salmeterol. The nurse would expect this drug to be administered by which route? Intravenous Inhalation Oral Subcutaneous

Inhalation Explanation: Salmeterol is administered via inhalation. Reference: Chapter 55: Drugs Acting on the Lower Respiratory Tract - Page 965

While reviewing an older adult's medical record, the nurse notes that the patient has solar lentigo. he nurse interprets this as which of the following? Bright red moles Hypertrophied scar tissue Dark discoloration of the skin Liver spots

Liver spots Explanation: Solar lentigo refers to liver spots. Melasma refers to dark discoloration of the skin. Cherry angioma is used to describe a bright red mole. Keloid is used to denote hypertrophied scar tissue. Reference: Hinkle, J.L., and Cheever, K.H. Brunner & Suddarth's Textbook of Medical-Surgical Nursing, 14th ed., Philadelphia: Wolters Kluwer, 2018, Chapter 60: Assessment of Integumentary Function, Chart 60-1, p. 1794. Chapter 60: Assessment of Integumentary Function - Page 1794

A patient with a longstanding diagnosis of chronic renal failure has experienced a significant decline in urine output in recent days, prompting him to seek care at a local clinic. A nurse at the clinic has suggested to a colleague that the administration of a diuretic such as hydrochlorothiazide may improve the patient's urine output. How should the colleague best respond to this suggestion? "That would only work if he could come in twice a day to get it intravenously." "That would probably help, but we'd have to do blood work first." "Maybe, but hydrochlorothiazide affects the bladder more than the kidneys." "Actually, patients with renal failure usually can't take hydrochlorothiazide."

"Actually, patients with renal failure usually can't take hydrochlorothiazide." Explanation: Renal disease and severe renal impairment contraindicate the use of hydrochlorothiazide. HCTZ affects the kidneys, not the bladder, and is not administered intravenously. Reference: Chapter 51: Diuretic Agents - Page 896

A nurse is caring for a client with heart failure. The nurse knows that the client has left-sided heart failure when the client makes which statement? "My feet are bigger than normal." "My pants don't fit around my waist." "I sleep on three pillows each night." "I don't have the same appetite I used to."

"I sleep on three pillows each night." Explanation: Orthopnea is a classic sign of left-sided heart failure. The client commonly sleeps on several pillows at night to help facilitate breathing. Swollen feet, ascites, and anorexia are signs of right-sided heart failure. Reference: Hinkle, J.L., and Cheever, K.H. Brunner & Suddarth's Textbook of Medical-Surgical Nursing, 14th ed., Philadelphia: Wolters Kluwer, 2018, Chapter 29: Management of Patients With Complications from Heart Disease, Left-Sided Heart Failure, p. 822. Chapter 29: Management of Patients With Complications from Heart Disease - Page 822

A client comes to the clinic reporting urinary symptoms. Which statement would most likely alert the nurse to suspect benign prostatic hyperplasia (BPH)? "I've had a fever and noticed I've been running to the bathroom more often." "I've had trouble getting started when I urinate, often straining to do so." "I've had some pain in my lower abdomen lately and felt a bit sick to my stomach." "I'm waking up at night to urinate and I've noticed some burning, too."

"I've had trouble getting started when I urinate, often straining to do so." Explanation: Symptoms that might alert the nurse to BPH include difficulty initiating urination and abdominal straining with urination. Although fever, urinary frequency, nocturia, pelvic pain, nausea, vomiting, and fatigue may be noted, they also may suggest other conditions such as urinary tract infection. Fever, nausea, vomiting, and fatigue are general symptoms that can accompany many conditions. Reference: Hinkle, J.L., and Cheever, K.H. Brunner & Suddarth's Textbook of Medical-Surgical Nursing, 14th ed., Philadelphia: Wolters Kluwer, 2018, Chapter 59: Assessment and Management of Patients with Male Reproductive Disorders, Clinical Manifestations, p. 1763. Chapter 59: Assessment and Management of Patients with Male Reproductive Disorders - Page 1763

Which client statement would lead the nurse to suspect that the client is experiencing bacterial conjunctivitis? "It feels like there is something stuck in my eye." "My eyes feel like they are on fire." "My eyes hurt when I'm in the bright sunlight." "My eyelids were stuck together this morning."

"My eyelids were stuck together this morning." Explanation: Burning, a sensation of a foreign body, and pain in bright light (photophobia) are signs and symptoms associated with any type of conjunctivitis. The drainage related to bacterial conjunctivitis is usually present in the morning, and the eyes may be difficult to open because of adhesions caused by the exudate. Reference: Hinkle, J.L., and Cheever, K.H. Brunner & Suddarth's Textbook of Medical-Surgical Nursing, 14th ed., Philadelphia: Wolters Kluwer, 2018, Chapter 63: Assessment and Management of Patients With Eye and Vision Disorders, Bacterial Conjunctivitis, p. 1906-1907. Chapter 63: Assessment and Management of Patients With Eye and Vision Disorders - Page

The nursing student asks the nurse how to tell the difference between ventricular tachycardia and ventricular fibrillation on an electrocardiogram strip. What is the best response? "Ventricular fibrillation is irregular with undulating waves and no QRS complex. Ventricular tachycardia is usually regular and fast, with wide QRS complexes." "The P-R interval will be prolonged in ventricular fibrillation, while in ventricular tachycardia the P-R interval is normal." "The two look very much alike; it is difficult to tell the difference." "The QRS complex in ventricular fibrillation is always narrow, while in ventricular tachycardia the QRS is of normal width."

"Ventricular fibrillation is irregular with undulating waves and no QRS complex. Ventricular tachycardia is usually regular and fast, with wide QRS complexes." Explanation: Ventricular fibrillation is irregular with undulating waves and no QRS complex, while ventricular tachycardia is usually regular and fast with wide QRS complexes. The rhythms look different on the electrocardiogram strip. The QRS is wide and bizarre or undefined in ventricular fibrillation. The P-R interval is not present in the ventricular dysrhythmias. Reference: Hinkle, J.L., and Cheever, K.H. Brunner & Suddarth's Textbook of Medical-Surgical Nursing, 14th ed., Philadelphia: Wolters Kluwer, 2018, Chapter 26: Management of Patients With Dysrhythmias and Conduction Problems, Ventricular Tachycardia, p. 730-731. Chapter 26: Management of Patients With Dysrhythmias and Conduction Problems - Page

An older female client calls and reports, "Another cyst in my private area. I had this years ago, but after 15 years it has returned. What should I do?" Which advice should the health care provider give to this client? "You should call and make an appointment. I would like to evaluate this further." "Try sitting in a warm bath to see if it helps." "I will call your pharmacy and order some antibiotic cream." "Are you allergic to any medication?"

"You should call and make an appointment. I would like to evaluate this further." Explanation: Because the Bartholin glands usually shrink during menopause, a vulvar growth in postmenopausal women is suspicious and should be evaluated for malignancy. Antibiotics and creams may be appropriate after physical assessment of the site, but these should not be the first line of action. If this cyst is at the Bartholin gland, local application of moist heat may be an appropriate course of action as well, following physical examination. Reference: Chapter 45: Disorders of the Female Reproductive System - Page 1129-1130

A client has suffered a deep partial-thickness burn to the right arm from a high-voltage source of energy that was not turned off while working on it. What is the priority nursing intervention in the acute phase of care? Monitor urine output once a shift. A cardiac monitor should be used for at least 24 hours to anticipate the potential for cardiac dysrhythmias. Infuse dextrose and water at 50 mL per hour to avoid overload of the circulatory system Initiate an antibiotic within 3 hours of the injury.

A cardiac monitor should be used for at least 24 hours to anticipate the potential for cardiac dysrhythmias. Explanation: A client with electrical burns based on energy and potential damage to the heart needs cardiac monitoring. Dextrose is not useful for fluid volume expansion and infection would occur much later. Urine output needs hourly monitoring based on myoglobin release.

A patient is scheduled for a Billroth I procedure for ulcer management. What does the nurse understand will occur when this procedure is performed? The antral portion of the stomach is removed and a vagotomy is performed. A partial gastrectomy is performed with anastomosis of the stomach segment to the duodenum. The vagus nerve is cut and gastric drainage is established. A sectioned portion of the stomach is joined to the jejunum.

A partial gastrectomy is performed with anastomosis of the stomach segment to the duodenum. Explanation: A Billroth I procedure involves removal of the lower portion of the antrum of the stomach (which contains the cells that secrete gastrin) as well as a small portion of the duodenum and pylorus. The remaining segment is anastomosed to the duodenum. A vagotomy severs the vagus nerve; a Billroth I procedure may be performed in conjunction with a vagotomy. If the remaining part of the stomach is anastomosed to the jejunum, the procedure is a Billroth II. Reference: Hinkle, J.L., and Cheever, K.H. Brunner & Suddarth's Textbook of Medical-Surgical Nursing, 14th ed., Philadelphia: Wolters Kluwer, 2018, Chapter 46: Management of Patients With Gastric and Duodenal Disorders, Table 46-4, p. 1300. Chapter 46: Management of Patients With Gastric and Duodenal Disorders - Page 1300

The nurse is preparing to administer a chemotherapy infusion to a client with esophageal cancer. The client has an implanted port that was last accessed 7 days ago. The insertion site is clean and dry and without erythema. Which is the appropriate action by the nurse? Access the port using a Huber needle. Access the port using a coring needle. Contact healthcare provider and write an incident report. Insert a peripheral line for the infusion.

Access the port using a Huber needle. Explanation: Implanted ports are designed for intermittent access; for the port to be last accessed 7 days ago is within accepted standards. There are no signs or symptoms of an infection at the site. Therefore, the nurse does not need to contact the healthcare provider or complete an incident report. The nurse should access the port with a Huber needle, which is designed for use in implanted ports. A coring needle will damage the implanted port. As the port is without signs and symptoms of complications, it should be utilized for the chemotherapy versus a peripheral line.

A patient tells the nurse that it feels like food is "sticking" in the lower portion of the esophagus. What motility disorder does the nurse suspect these symptoms indicate? Hiatal hernia Gastroesophageal reflex disease Diffuse spasm Achalasia

Achalasia Explanation: Achalasia is absent or ineffective peristalsis of the distal esophagus accompanied by failure of the esophageal sphincter to relax in response to swallowing. Narrowing of the esophagus just above the stomach results in a gradually increasing dilation of the esophagus in the upper chest. The main symptom is difficulty in swallowing both liquids and solids. The patient has a sensation of food sticking in the lower portion of the esophagus. Reference: Hinkle, J.L., and Cheever, K.H. Brunner & Suddarth's Textbook of Medical-Surgical Nursing, 14th ed., Philadelphia: Wolters Kluwer, 2018, Chapter 45: Management of Patients With Oral and Esophageal Disorders, Clinical Manifestations, p. 1279. Chapter 45: Management of Patients With Oral and Esophageal Disorders - Page 1279 Add a Note

Following surgery for adenocarcinoma, the client learns the tumor stage is T3,N1,M0. What treatment mode should the nurse anticipate? Repeat biopsy is needed before treatment begins. Palliative care is likely. No further treatment is indicated. Adjuvant therapy is likely.

Adjuvant therapy is likely. Explanation: T3 indicates a large tumor size, with N1 indicating regional lymph node involvement so treatment is needed. A T3 tumor must have its size reduced with adjuncts like chemotherapy and radiation. Although M0 suggest no metastasis, following with adjuvant (chemotherapy or radiation therapy) treatment is indicated to prevent the spread of cancer outside the lymph to other organs. The tumor stage IV wound be indicative of palliative care. A repeated biopsy is not needed until after treatment is completed. Reference: Hinkle, J.L., and Cheever, K.H. Brunner & Suddarth's Textbook of Medical-Surgical Nursing, 14th ed., Philadelphia: Wolters Kluwer, 2018, Chapter 15: Management of Patients with Oncologic Disorders, Chart 15-3: TNM Classification System, p. 335. Chapter 15: Management of Patients with Oncologic Disorders - Page 335

The new nurse is having difficulty managing the time required to care for a group of complex clients and is several hours behind in completing nursing interventions. Which intervention should the nurse complete first? Complete a medication reconciliation form on a client who has recently been admitted to the hospital. Obtain discharge orders for a client who is ready to be transferred to a long-term nursing facility. Administer a dose of digoxin that is two hours behind schedule. Perform a dressing change to an abdominal abscess that is three hours behind schedule.

Administer a dose of digoxin that is two hours behind schedule. Explanation: The first step in time management is to determine which tasks are priority. Digoxin is a critical client medication and therefore takes priority over the other options. Dressing changes, discharge orders, and completing facility forms can be delayed until critical tasks are complete. Reference: Chapter 10: Leadership, Managing and Delegating - Page 226

A client presents to the community health office experiencing rapidly increasing symptoms of anaphylactic shock. Which nursing action would be completed first? Obtain a health history. Obtain the name and information of the allergic substance. Administer an epinephrine injection as ordered by the health care provider. Call 911.

Administer an epinephrine injection as ordered by the health care provider. Explanation: The key words in the question are "increasing symptoms." The first action of the nurse is to administer an epinephrine injection to abort the rapidly increasing symptoms. Next, the nurse will call 911. Reference: Hinkle, J.L., and Cheever, K.H. Brunner & Suddarth's Textbook of Medical-Surgical Nursing, 14th ed., Philadelphia: Wolters Kluwer, 2018, Chapter 14: Shock and Multiple Organ Dysfunction Syndrome, Medical Management, p. 319. Chapter 14: Shock and Multiple Organ Dysfunction Syndrome - Page 319

The nurse is told by a client that the client is having suicidal thoughts. Which intervention has lowest priority? Administering a mental status exam to assess for psychosis Assessing the client for past history of suicidal attempts Maintaining a safe, secure environment Determining the client's concerns and if the client has a plan

Administering a mental status exam to assess for psychosis Explanation: About 50% to 80% of people who commit suicide have previously attempted suicide; the more violent and lethal the plan, the higher the potential for suicide. Assessment of past attempts and current plan (not psychosis) as well as maintaining client safety would be priorities. Maintaining a safe, secure environment is an important intervention by the nurse to prevent a suicide attempt. Reference: Chapter 17: Mood Disorders and Suicide - Page 319

A client is admitted with acute osteomyelitis that developed after an open fracture of the right femur. When planning this client's care, the nurse should anticipate which measure? Withholding all oral intake Instructing the client to ambulate twice daily Administering large doses of I.V. antibiotics as ordered Administering large doses of oral antibiotics as ordered

Administering large doses of I.V. antibiotics as ordered Explanation: Treatment of acute osteomyelitis includes large doses of I.V. antibiotics (after blood cultures identify the infecting organism). Surgical drainage may be indicated, and the affected bone is immobilized. The client usually requires I.V. fluids to maintain hydration, but oral intake isn't necessarily prohibited. Reference: Hinkle, J.L., and Cheever, K.H. Brunner & Suddarth's Textbook of Medical-Surgical Nursing, 14th ed., Philadelphia: Wolters Kluwer, 2018, Chapter 41: Management of Patients With Musculoskeletal Disorders, Osteomyelitis and Wound Infections, p. 1182. Chapter 41: Management of Patients With Musculoskeletal Disorders - Page 1182

A nurse is working with a patient to establish a bowel training program. Based on the nurse's understanding of bowel function, the nurse would suggest planning for bowel evacuation at which time? After breakfast Before bed Upon arising Around lunchtime

After breakfast Explanation: Natural gastrocolic and duodenocolic reflexes occur about 30 minutes after a meal; therefore, after breakfast is one of the best times to plan for bowel evacuation. Reference: Hinkle, J. L., Cheever, K. H. Brunner & Suddarth's Textbook of Medical-Surgical Nursing , 14th ed Philadelphia: Wolters Kluwer Health/Lippincott Williams & Wilkins, 2018, Chapter 10: Principles and Practices of Rehabilitation, p. 188. Chapter 10: Principles and Practices of Rehabilitation - Page 188 Add a Note

When assisting the patient to interpret a negative HIV test result, the nurse informs the patient that the results mean which of the following? Antibodies to HIV are not present in his blood. Antibodies to HIV are present in his blood. He has not been infected with HIV. He is immune to HIV.

Antibodies to HIV are not present in his blood. Explanation: A negative test result indicates that antibodies to HIV are not present in the blood at the time the blood sample for the test is drawn. A negative test result should be interpreted as demonstrating that if infected, the body has not produced antibodies (which take from 3 weeks to 6 months or longer). Therefore, subsequent testing of an at-risk patient must be encouraged. The test result does not mean that the patient is immune to the virus, nor does it mean that the patient is not infected. It just means that the body may not have produced antibodies yet. When antibodies to HIV are detected in the blood, the test is interpreted as positive. Reference: Hinkle, J.L., and Cheever, K.H. Brunner & Suddarth's Textbook of Medical-Surgical Nursing, 14th ed., Philadelphia: Wolters Kluwer, 2018, Chapter 36: Management of Patients With Immune Deficiency Disorders, Chart 36-7, p. 1032. Chapter 36: Management of Patients With Immune Deficiency Disorders - Page 1032

A client prescribed pain medication around the clock experiences pain 1 hour before the next dose of the pain medication is due. Which is the most appropriate action by the nurse? Tell the client he or she will have to wait for 1 hour. Administer the next dose of the pain medication. Assess the client for signs of opioid addiction. Assess for medication prescription for breakthrough pain.

Assess for medication prescription for breakthrough pain. Explanation: Breakthrough pain is a temporary flare-up of moderate to severe pain that occurs even when the client is taking pain medication around the clock. It can occur before the next dose of analgesic is due (end of dose pain). It is treated most effectively with supplemental doses of a short-acting opioid taken on an "as needed basis." Therefore, the nurse should check for a prescription for breakthrough pain medication. Telling the client that he or she has to wait is not a therapeutic action by the nurse. Administering the next dose of pain medication is a violation of nursing practice and does not follow the standard of care. The nurse needs to assess for the therapeutic effects of the pain medication and not opioid addiction. Reference: Chapter 35: Comfort and Pain Management - Page 1259

One hour after receiving pain medication, a postoperative client reports intense pain. What is the nurse's appropriate first action? Assess the client to determine the cause of the pain. Discuss the frequency of pain medication administration with the client. Consult with the physician for additional pain medication. Assist the client to reposition and splint the incision.

Assess the client to determine the cause of the pain. Explanation: One hour after administering pain medication, the nurse would expect the client to be relieved of pain. A new report of intense pain might signal a complication and requires a thorough assessment. The nurse might request an order for additional pain medication, but only after a thorough assessment. Telling the client how often medication can be received does not help relieve the client's pain. Repositioning and splinting the incision are interventions that the nurse might perform, but only after determining the cause of the pain. Reference: Chapter 17: Implementing - Page 417

A client newly diagnosed with otitis media reports that the pain and pressure in the ear has suddenly disappeared. What is the best action by the nurse? Assess the tympanic membrane. Document the effectiveness of medications. Irrigate the ear. Educate the client on the therapeutic effects of medications.

Assess the tympanic membrane. Explanation: A client diagnosed with otitis media who feels sudden relief of pain and/or pressure should be assessed for a tympanic membrane rupture. Educating the client on the therapeutic effects of medications is appropriate for newly diagnosed otitis media, but it does not address the sudden disappearance of pain and pressure. Because the medication usually takes 48 to 72 hours to be effective, documenting the medication as effective would be inappropriate. It is not necessary to irrigate an ear with otitis media. Reference: Hinkle, J.L., and Cheever, K.H. Brunner & Suddarth's Textbook of Medical-Surgical Nursing, 14th ed., Philadelphia: Wolters Kluwer, 2018, Chapter 64: Assessment and Management of Patients With Hearing and Balance Disorders, Table 64-3, p. 1928. Chapter 64: Assessment and Management of Patients With Hearing and Balance Disorders - Page 1928

A client in end-stage renal disease is prescribed epoetin alfa and oral iron supplements. Before administering the next dose of epoetin alfa and oral iron supplement, what is the priority action taken by the nurse? Questions the administration of both medications Assesses the hemoglobin level Ensures the client has completed dialysis treatment Holds the epoetin alfa if the BUN is elevated

Assesses the hemoglobin level Explanation: Erythropoietin (epoetin alfa [Epogen]) with oral iron supplements can raise hematocrit levels in the client with end-stage renal disease. The nurse should check the hemoglobin prior to administration of erythropoietin, because too high a hemoglobin level can put the client at risk for heart failure, myocardial infarction, and cerebrovascular accident. Erythropoietin may be administered during dialysis treatments. The BUN will be elevated in the client with end-stage renal disease. Reference: Hinkle, J.L., and Cheever, K.H. Brunner & Suddarth's Textbook of Medical-Surgical Nursing, 13th ed. Philadelphia: Lippincott Williams & Wilkins, 2014, Chapter 32: Assessment of Hematologic Function and Treatment Modalities, p. 897. Chapter 32: Assessment of Hematologic Function and Treatment Modalities - Page 897

A female client is diagnosed with breast abscess. She would like to continue to breast-feed her newborn. Which of the following would be most appropriate in this situation? Reduce the frequency of removing and reapplying the dressings. Instruct the client to wear a tight-fitting bra. Assist the client to pump the breasts to remove breast milk. Encourage the client to include protein content in the diet.

Assist the client to pump the breasts to remove breast milk. Explanation: The nurse should help the client pump the breasts and remove breast milk to prevent engorgement. Because the client has decided to continue breastfeeding, the client should wear a loose-fitting bra. Including protein content in the diet would be unrelated to the client's current situation. Frequency of dressing changes does not play a role in the intervention. Reference: Hinkle, J.L., & Cheever, K.H., Brunner & Suddarth's Textbook of Medical-Surgical Nursing, 14th ed., Philadelphia, Wolters Kluwer, 2018.

The nurse is evaluating bloodwork results of a client with cancer who is receiving chemotherapy. The client's platelet count is 60,000/mm3. Which is an appropriate nursing action? Providing a razor so the client can shave Taking the client's temperature rectally Avoiding the use of products containing aspirin Providing commercial mouthwash to the client

Avoiding the use of products containing aspirin Explanation: Clients with a platelet count of 60,000/mm3 are at mild risk for bleeding. Appropriate nursing interventions include avoiding the use of products such as aspirin that may interfere with the client's clotting systems; avoiding taking temperature rectally and administering suppositories; providing the client with an electric shaver for shaving; and avoiding commercial mouthwashes because of their potential to dry out oral mucosa, which can lead to cracking and bleeding. Reference:

The nurse is to obtain a stool specimen from a client who reported that he is taking iron supplements. The nurse would expect the stool to be which color? Black Red Dark brown Green

Black Explanation: Ingestion of iron can cause the stool to turn black. Meat protein causes stool to appear dark brown. Ingestion of large amounts of spinach may turn stool green while ingestion of carrots and beets may cause stool to turn red. Reference: Hinkle, J.L., and Cheever, K.H. Brunner & Suddarth's Textbook of Medical-Surgical Nursing, 14th ed., Philadelphia: Wolters Kluwer, 2018, Chapter 43: Assessment of Digestive and Gastrointestinal Function, Table 43-4, p. 1230. Chapter 43: Assessment of Digestive and Gastrointestinal Function - Page 1230

Which of the following is a strong positive reaction to patch testing? Select all that apply. Pain Itching Ulceration Blisters Redness

Blisters Pain Ulceration Explanation: Blisters, pain, and ulceration indicate a strong positive reaction to patch testing. The development of redness, fine elevations, or itching is considered a weak positive reaction. Reference: Hinkle, J.L., and Cheever, K.H. Brunner & Suddarth's Textbook of Medical-Surgical Nursing, 14th ed., Philadelphia: Wolters Kluwer, 2018, Chapter 60: Assessment of Integumentary Function, Patch Testing, p. 1804. Chapter 60: Assessment of Integumentary Function - Page 1804

Over the past 2 months, a client has been receiving treatment for multiple ear infections and tonsillitis. The client reports a curdy white vaginal discharge and burning with urination. What is the most likely cause of these symptoms? None of the options is correct. Candida albicans Trichomonas vaginalis Gardnerella vaginalis

Candida albicans Explanation: Candida albicans presents with a thick, curdy white discharge, accompanied by a strong odor and burning with urination. Trichomonas vaginalis presents with a foamy, yellow-white discharge, accompanied by a foul odor and severe itching. Gardnerella vaginalis presents with a watery, gray-white discharge, accompanied by a fishy odor and more discharge after intercourse. Reference: Hinkle, J.L., and Cheever, K.H. Brunner & Suddarth's Textbook of Medical-Surgical Nursing, 14th ed., Philadelphia: Wolters Kluwer, 2018, Chapter 57: Management of Patients With Female Reproductive Disorders, p. 1687. Chapter 57: Management of Patients With Female Reproductive Disorders - Page 1687

While the nurse is performing a physical assessment, the client reports numbness, tingling, and pain when the nurse percusses lightly over the median nerve. What should this assessment indicate to the nurse? Impingement syndrome Carpal tunnel syndrome Morton's neuroma Dupuytren's contracture

Carpal tunnel syndrome Explanation: Reference: Hinkle, J.L., and Cheever, K.H. Brunner & Suddarth's Textbook of Medical-Surgical Nursing, 14th ed., Philadelphia: Wolters Kluwer, 2018, Chapter 41: Management of Patients With Musculoskeletal Disorders, Carpal Tunnel Syndrome, p. 1164. Chapter 41: Management of Patients With Musculoskeletal Disorders - Page 1164

A nurse is obtaining family history from a patient. Which of the following would be LEAST helpful to use when documenting this information? Family tree Genogram Checklist Pedigree

Checklist Explanation: When recording the family history, family trees, genograms, and pedigrees are most helpful. Checklists are helpful when documenting a review of systems. Reference: Hinkle, J. L., Cheever, K. H. Brunner & Suddarth's Textbook of Medical-Surgical Nursing , 14th ed Philadelphia: Wolters Kluwer Health/Lippincott Williams & Wilkins, 2018, Chapter 5: Adult Health and Nutritional Assessment, p. 66. Chapter 5: Adult Health and Nutritional Assessment - Page 66 Add a Note

A client who has been taking a statin has seen an improvement in his cholesterol laboratory values; however, the low-density lipoprotein remains elevated. What medication will be added to the medical regime? Vitamin D Calcium carbonate Digoxin Cholestyramine

Cholestyramine Explanation: Cholestyramine is administered to clients to reduce LDL cholesterol in clients who are already taking a statin drug. Digoxin is not given to lower LDL cholesterol. Vitamin D is not given to lower LDL cholesterol. Calcium carbonate is not given to lower LDL cholesterol. Reference: Chapter 47: Lipid-Lowering Agents - Page 814

A client is admitted to the hospital with a diagnosis of pneumonia. The client informs the nurse that he has several drug allergies. The physician has ordered an antibiotic as well as several other medications for cough and fever. What should the nurse do prior to administering the medications? Call the pharmacy and let them know the client has several drug allergies. Give the client one medicine at a time and observe for allergic reactions. Administer the medications that the physician ordered. Consult drug references to make sure the medicines do not contain substances which the client is hypersensitive.

Consult drug references to make sure the medicines do not contain substances which the client is hypersensitive. Explanation: Clear identification of any substances to which the client is allergic is essential. The nurse must consult drug references to verify that prescribed medications do not contain substances to which the client is hypersensitive. Administering the medications or giving one at a time may cause the client to have an allergic reaction. The nurse may call the pharmacy but still maintains responsibility for the medications administered. Reference: Hinkle, J.L., and Cheever, K.H. Brunner & Suddarth's Textbook of Medical-Surgical Nursing, 14th ed., Philadelphia: Wolters Kluwer, 2018, Chapter 35: Assessment of Immune Function, Assessment of the Immune System, p. 1014. Chapter 35: Assessment of Immune Function - Page 1014

A client has a cast applied to the leg for treatment of a tibia fracture and also has a wound on the leg that requires dressing changes due to drainage. For what should the nurse prepare the client? Removal of the cast Cutting of a bivalve cast Cutting a cast window Insertion of an external fixator

Cutting a cast window Explanation: After the cast dries, a cast window, or opening, may be cut. This usually is done when the client reports discomfort under the cast or has a wound that requires a dressing change. The window permits direct inspection of the skin, a means to check the pulse in a casted arm or leg, or a way to change a dressing. A bivalve cast is when the cast is cut in two if the leg swells or if the client is being weaned from a cast, when a sharp x-ray is needed, or as a splint for immobilizing painful joints when a client has arthritis. The cast should not be removed due to the instability of a fracture. The client's condition does not indicate an external fixator is required. Reference: Hinkle, J.L., and Cheever, K.H. Brunner & Suddarth's Textbook of Medical-Surgical Nursing, 14th ed., Philadelphia: Wolters Kluwer, 2018, Chapter 40: Musculoskeletal Care Modalities, Pressure Ulcers, p. 1135. Chapter 40: Musculoskeletal Care Modalities - Page 1135

A client has undergone diagnostic testing that involved the insertion of a lighted tube with a telescopic lens. The nurse identifies this test as which of the following? Excretory urogram Cystoscopy Intravenous pyelography Renal angiography

Cystoscopy Explanation: Reference: Hinkle, J.L., and Cheever, K.H. Brunner & Suddarth's Textbook of Medical-Surgical Nursing, 14th ed., Philadelphia: Wolters Kluwer, 2018, Chapter 53: Assessment of Kidney and Urinary Function, Urologic Endoscopic Procedures, p. 1564. Chapter 53: Assessment of Kidney and Urinary Function - Page 1564

When circulatory shock occurs, there is massive vasodilation causing pooling of the blood in the periphery of the body. An ICU nurse caring for a patient in circulatory shock knows that the pooling of blood in the periphery leads to: Decreased heart rate Decreased venous return Increased stroke volume Increased cardiac output

Decreased venous return Explanation: Pooling of blood in the periphery results in decreased venous return. Decreased venous return results in decreased stroke volume and decreased cardiac output. Decreased cardiac output, in turn, causes decreased blood pressure and, ultimately, decreased tissue perfusion. The heart rate increases in an attempt to meet the demands of the body Reference: Chapter 43: Drugs Affecting Blood Pressure - Page 745

A client comes to the emergency department complaining of pain in the right leg. When obtaining the history, the nurse learns that the client has a history of obesity and hypertension. Based on this information the nurse anticipates the client having which musculoskeletal disorder? Degenerative joint disease Muscular dystrophy Scoliosis Paget's disease

Degenerative joint disease Explanation: Obesity predisposes the client to degenerative joint disease. Obesity isn't a predisposing factor for muscular dystrophy, scoliosis, or Paget's disease. Reference: Hinkle, J.L., and Cheever, K.H. Brunner & Suddarth's Textbook of Medical-Surgical Nursing, 14th ed., Philadelphia: Wolters Kluwer, 2018, Chapter 38: Assessment and Management of Patients With Rheumatic Disorders, Osteoarthritis (Degenerative Joint Disease), p. 1104. Chapter 38: Assessment and Management of Patients With Rheumatic Disorders - Page 1104

The nurse is changing the dressing of a client who is 4 days postoperative with an abdominal wound. The nurse has changed this dressing daily since surgery. Today, the nurse notes increased serosanguinous drainage, wound edges not approximated, and a ¼-inch (6 mm) gap at the lower end of the incision. The nurse concludes which of the following conditions exists? Hemorrhage Normal healing by primary intention. Evisceration Dehiscence

Dehiscence Explanation: Dehiscence is a disruption of the incision. Reference: Hinkle, J. L., Cheever, K. H. Brunner & Suddarth's Textbook of Medical-Surgical Nursing , 14th ed Philadelphia: Wolters Kluwer Health/Lippincott Williams & Wilkins, 2018, Chapter 19: Postoperative Nursing Management, p. 474. Chapter 19: Postoperative Nursing Management - Page 474

nurse is caring for an elderly bedridden adult. To prevent pressure ulcers, which intervention should the nurse include in the care plan? Slide the client, rather than lifting, when turning. Develop a written, individual turning schedule. Vigorously massage lotion over bony prominences. Turn and reposition the client at least once every 8 hours.

Develop a written, individual turning schedule. Explanation: A turning schedule sheet helps ensure that the client gets turned and, thus, helps prevent pressure ulcers. Turning should occur every 1 to 2 hours — not every 8 hours — for clients who are in bed for prolonged periods. The nurse should apply lotion to keep the skin moist but should avoid vigorous massage, which could damage capillaries. When moving the client, the nurse should lift — rather than slide — the client to avoid shearing. Reference: Hinkle, J. L., Cheever, K. H. Brunner & Suddarth's Textbook of Medical-Surgical Nursing , 14th ed Philadelphia: Wolters Kluwer Health/Lippincott Williams & Wilkins, 2018, Chapter 10: Principles and Practices of Rehabilitation, p. 183. Chapter 10: Principles and Practices of Rehabilitation - Page 183

A client is prescribed risperidone for the treatment of schizophrenia. The client is voiding three times each night and is always thirsty. Based on the adverse effects of risperidone, what should the nurse suspect is triggering the client's reported polyuria and polydipsia? Urinary tract infection Renal calculi Hyperthyroidism Diabetes mellitus

Diabetes mellitus Explanation: The development of polyuria and polydipsia is indicative of diabetes mellitus. Risperidone has been associated with weight gain, diabetes, and dyslipidemia. Adverse effects of risperidone do not include urinary tract infection, renal calculus, or the development of hyperthyroidism. Reference: Chapter 22: Psychotherapeutic Agents - Page 380

The nurse cares for a client with gallstones that need to be removed but is not a surgical candidate or endoscopic candidate. What procedure does the nurse recognize as being a possible treatment option for the client? Transnasal biliary catheter insertion T-tube insertion ESWL ERCP

ESWL Explanation: Extracorporeal shock wave therapy, (ESWL) also known as lithotripsy, uses shock waves to break up gallstones for their removal and does not involve surgery or endoscopy. The other answer choices are performed by endoscopy. Reference: Hinkle, J.L., and Cheever, K.H. Brunner & Suddarth's Textbook of Medical-Surgical Nursing, 14th ed., Philadelphia: Wolters Kluwer, 2018, Chapter 50: Assessment and Management of Patients With Biliary Disorders, p. 1435. Chapter 50: Assessment and Management of Patients With Biliary Disorders - Page 1435

A nursing instructor is discussing the causes of the increasing number of people with chronic conditions. Which of the following would the nurse correctly identify as a cause? An increased mortality rate from infectious diseases Shorter lifespans Early detection and treatment of diseases Lowered stress and increased physical activity lifestyles

Early detection and treatment of diseases Explanation: Improved screening and diagnostic procedures enable early detection and treatment of diseases, resulting in improved outcomes of management of cancer and other disorders. Lifestyle factors, such as smoking, chronic stress, and sedentary lifestyle, increase the risk of chronic health problems such as respiratory disease, hypertension, cardiovascular disease, and obesity. Longer lifespans are because of advances in technology and pharmacology, and a decrease in mortality from infectious diseases. Reference: Hinkle, J. L., Cheever, K. H. Brunner & Suddarth's Textbook of Medical-Surgical Nursing , 14th ed Philadelphia: Wolters Kluwer Health/Lippincott Williams & Wilkins, 2018, Chapter 9: Chronic Illness and Disability, p. 143. Chapter 9: Chronic Illness and Disability - Page 143

A nurse should perform which intervention for a client with Cushing's syndrome? Suggest a high-carbohydrate, low-protein diet. Explain that the client's physical changes are a result of excessive corticosteroids. Offer clothing or bedding that's cool and comfortable. Explain the rationale for increasing salt and fluid intake in times of illness, increased stress, and very hot weather.

Explain that the client's physical changes are a result of excessive corticosteroids. Explanation: The nurse should explain to the client that Cushing's syndrome causes physical changes related to excessive corticosteroids. Clients with hyperthyroidism, not Cushing's syndrome, are heat intolerant and must have cool clothing and bedding. Clients with Cushing's syndrome should have a high-protein, not low-protein, diet. Clients with Addison's disease must increase sodium intake and fluid intake in times of stress of prevent hypotension.

A nurse is performing an otoscopic examination on a client. Which finding would the nurse document as abnormal? External auditory canal erythema Manubrium superior to the umbo Tympanic membrane pearly gray Umbo in the center of the tympanic membrane

External auditory canal erythema Explanation: An erythematous external auditory canal would be considered an abnormal finding. The tympanic membrane is normally pearly gray and translucent. The umbo, which is located in the center of the eardrum, extends from the superior manubrium. Reference: Hinkle, J.L., and Cheever, K.H. Brunner & Suddarth's Textbook of Medical-Surgical Nursing, 14th ed., Philadelphia: Wolters Kluwer, 2018, Chapter 64: Assessment and Management of Patients With Hearing and Balance Disorders, Clinical Manifestations, p. 1926. Chapter 64: Assessment and Management of Patients With Hearing and Balance Disorders - Page 1926

The client is receiving a vesicant antineoplastic for treatment of cancer. Which assessment finding would require the nurse to take immediate action? Stomatitis Extravasation Bone pain Nausea and vomiting

Extravasation Explanation: The nurse needs to monitor IV administration of antineoplastics (especially vesicants) to prevent tissue necrosis to blood vessels, skin, muscles, and nerves. Stomatitis, nausea/vomiting, and bone pain can be symptoms of the disease process or treatment mode but does not require immediate action. Reference: Hinkle, J.L., and Cheever, K.H. Brunner & Suddarth's Textbook of Medical-Surgical Nursing, 14th ed., Philadelphia: Wolters Kluwer, 2018, Chapter 15: Management of Patients with Oncologic Disorders, Extravasation, p. 342. Chapter 15: Management of Patients with Oncologic Disorders - Page 342

If untreated, squamous cell carcinoma of the external ear can spread through the temporal bone causing which of the following? Facial nerve paralysis Diplopia Nystagmus Motor impairment

Facial nerve paralysis Explanation: If untreated, squamous cell carcinomas of the ear can spread through the temporal bone, causing facial nerve paralysis and hearing loss. Reference: Hinkle, J.L., and Cheever, K.H. Brunner & Suddarth's Textbook of Medical-Surgical Nursing, 14th ed., Philadelphia: Wolters Kluwer, 2018, Chapter 64: Assessment and Management of Patients With Hearing and Balance Disorders, Masses of the External Ear, p. 1927. Chapter 64: Assessment and Management of Patients With Hearing and Balance Disorders - Page 1927

A toddler who is beginning to walk has fallen and hit his head on the corner of a low table. The caregiver has been unable to stop the bleeding and brings the child to the pediatric clinic. The nurse is gathering data during the admission process and notes several bruises and swollen joints. A diagnosis of hemophilia is confirmed. This child most likely has a deficiency of which blood factor? Factor X Factor VIII Factor V Factor XIII

Factor VIII Explanation: The most common types of hemophilia are factor VIII deficiency and factor IX deficiency, which are inherited as sex-linked recessive traits, with transmission to male offspring by carrier females. Reference: Ricci, S.S., Kyle, T., Carman, S. Maternity and Pediatric Nursing, 3rd ed. Philadelphia: Wolters Kluwer Health, 2017, Chapter 46: Nursing Care of the Child With an Alteration in Cellular Regulation/Hematologic or Neoplastic Disorder, p. 1815. Chapter 46: Nursing Care of the Child With an Alteration in Cellular Regulation/Hematologic or Neoplastic Disorder - Page 1815

A toddler who is beginning to walk has fallen and hit his head on the corner of a low table. The caregiver has been unable to stop the bleeding and brings the child to the pediatric clinic. The nurse is gathering data during the admission process and notes several bruises and swollen joints. A diagnosis of hemophilia is confirmed. This child most likely has a deficiency of which blood factor? Factor V Factor VIII Factor X Factor XIII

Factor VIII Explanation: The most common types of hemophilia are factor VIII deficiency and factor IX deficiency, which are inherited as sex-linked recessive traits, with transmission to male offspring by carrier females. Reference: Ricci, S.S., Kyle, T., Carman, S. Maternity and Pediatric Nursing, 3rd ed. Philadelphia: Wolters Kluwer Health, 2017, Chapter 46: Nursing Care of the Child With an Alteration in Cellular Regulation/Hematologic or Neoplastic Disorder, p. 1815. Chapter 46: Nursing Care of the Child With an Alteration in Cellular Regulation/Hematologic or Neoplastic Disorder - Page 1815 Add a Note

The nursing assessment of the postoperative client reveals an incision that is well-approximated with sutures intact, minimal redness and edema, and absence of drainage. The nurse recognizes the wound is healing by: Second intention Third intention First intention Granulation

First intention Explanation: First-intention healing is characterized by a closed incision with little tissue reaction and the absence of signs and symptoms of infection. Reference: Hinkle, J. L., Cheever, K. H. Brunner & Suddarth's Textbook of Medical-Surgical Nursing , 14th ed Philadelphia: Wolters Kluwer Health/Lippincott Williams & Wilkins, 2018, Chapter 19: Postoperative Nursing Management, p. 468. Chapter 19: Postoperative Nursing Management - Page 468

The nurse is assessing an acutely ill patient. When prioritizing the patient's care, the nurse should recognize that the patient is at risk for hypovolemic shock when: Cardiac output is increased. Blood pressure increases. Fluid circulating in the blood vessels decreases. Pulse is fast and bounding.

Fluid circulating in the blood vessels decreases. Explanation: Hypovolemic shock is characterized by a decrease in intravascular volume. Cardiac output is decreased, blood pressure decreases, and the pulse is fast but weak.

The nurse is administering furosemide I.V. push through a Groshong catheter. What is the appropriate action by the nurse after the medication is administered? Flush with normal saline. Flush with the primary I.V. solution. Flush with heparin then normal saline. Flush with heparin.

Flush with normal saline. Explanation: The nurse should flush the line with normal saline after administering the medication. Due to the Groshong one-way valve, there is no need to follow the normal saline flush with a heparin flush or to flush with heparin. Flushing with the primary I.V. solution is not an appropriate answer because the line should always be flushed with a saline flush to ensure that the one-way valve opens and closes appropriately to prevent backflow.

The nurse is caring for a client who has premature ventricular contractions. What sign or symptom is observed in this client? Hypotension Nausea Fever Fluttering

Fluttering Explanation: Premature ventricular contractions usually cause a flip-flop sensation in the chest, sometimes described as "fluttering." Associated signs and symptoms include pallor, nervousness, sweating, and faintness. Symptoms of premature ventricular contractions are not nausea, hypotension, and fever. Reference: Hinkle, J.L., & Cheever, K.H., Brunner & Suddarth's Textbook of Medical-Surgical Nursing, 14th ed., Philadelphia, Wolters Kluwer, 2018.

When the nurse is caring for a patient with acute pancreatitis, what intervention can be provided in order to prevent atelectasis and prevent pooling of respiratory secretions? Frequent changes of positions Perform chest physiotherapy Suction the patient every 4 hours Placing the patient in the prone position

Frequent changes of positions Explanation: Frequent changes of position are necessary to prevent atelectasis and pooling of respiratory secretions. Reference: Hinkle, J.L., and Cheever, K.H. Brunner & Suddarth's Textbook of Medical-Surgical Nursing, 14th ed., Philadelphia: Wolters Kluwer, 2018, Chapter 50: Assessment and Management of Patients With Biliary Disorders, Improving Breathing Pattern, p. 1443. Chapter 50: Assessment and Management of Patients With Biliary Disorders - Page 1443 Add a Note

A patient is receiving omalizumab to treat allergic asthma, which is not relieved by inhaled corticosteroids. Which nursing intervention is appropriate with each dose administration of omalizumab? Avoid high-fat foods Administer a corticosteroid Assess for cancer Have epinephrine available

Have epinephrine available Explanation: Health care providers should have medications and supplies available for treatment of anaphylaxis, should observe patients for at least 2 hours after an injection, and should teach patients how to self-administer emergency treatment. The patient will not require assessment for cancer with every dose administration. The patient will not need to avoid high-fat foods. The patient will not receive corticosteroid agents. Reference: Chapter 17: Immune Modulators - Page 303-304 Add a Note

A client with congestive heart failure is admitted to the hospital after reporting shortness of breath. How should the nurse position the client in order to decrease preload? Supine with arms elevated on pillows above the level of the heart Head of the bed elevated 45 degrees and lower arms supported by pillows Head of the bed elevated 30 degrees and legs elevated on pillows Prone with legs elevated on pillows

Head of the bed elevated 45 degrees and lower arms supported by pillows Explanation: Preload refers to the degree of stretch of the ventricular cardiac muscle fibers at the end of diastole. The client is positioned or taught how to assume a position that facilitates breathing. The number of pillows may be increased, the head of the bed may be elevated, or the client may sit in a recliner. In these positions, the venous return to the heart (preload) is reduced, pulmonary congestion is alleviated, and pressure on the diaphragm is minimized. The lower arms are supported with pillows to eliminate the fatigue caused by the pull of the client's weight on the shoulder muscles. Reference: Hinkle, J.L., and Cheever, K.H. Brunner & Suddarth's Textbook of Medical-Surgical Nursing, 14th ed., Philadelphia: Wolters Kluwer, 2018, Chapter 29: Management of Patients With Complications from Heart Disease, p. 830. Chapter 29: Management of Patients With Complications from Heart Disease - Page 830 Add a Note

Which of the following indicates that a client with HIV has developed AIDS? Pain on standing and walking Severe fatigue at night Weight loss of 10 lb over 3 months Herpes simplex ulcer persisting for 2 months

Herpes simplex ulcer persisting for 2 months Explanation: A diagnosis of AIDS cannot be made until the person with HIV meets case criteria established by the Centers for Disease Control and Prevention. The immune system becomes compromised. The CD4 T-cell count drops below 200 cells and develops one of the opportunistic diseases, such as Pneumocystis carinii pneumonia, candidiasis, cytomegalovirus, or herpes simplex. Reference: Hinkle, J.L., and Cheever, K.H. Brunner & Suddarth's Textbook of Medical-Surgical Nursing, 14th ed., Philadelphia: Wolters Kluwer, 2018, Chapter 36: Management of Patients With Immune Deficiency Disorders, Integumentary Manifestations, p. 1041. Chapter 36: Management of Patients With Immune Deficiency Disorders - Page 1041 Add a Note

A client in the ED has a 5-cm thoracic aortic aneurysm that was discovered during a routine chest x-ray. When obtaining the client's history, which symptoms will it be most important for the nurse to ask about? Back or lumbar pain Hoarse voice and difficulty swallowing Abdominal swelling and tenderness Changes in bowel and bladder habits

Hoarse voice and difficulty swallowing Explanation: Symptoms are dyspnea, the result of pressure of the aneurysm sac against the trachea, a main bronchus, or the lung itself; cough, frequently paroxysmal and with a brassy quality; hoarseness, stridor, or weakness or complete loss of the voice (aphonia), resulting from pressure against the laryngeal nerve; and dysphagia (difficulty in swallowing) due to impingement of the aneurysm on the esophagus. Reference: Hinkle, J.L., and Cheever, K.H. Brunner & Suddarth's Textbook of Medical-Surgical Nursing, 14th ed., Philadelphia: Wolters Kluwer, 2018, Chapter 30: Assessment and Management of Patients With Vascular Disorders and Problems of Peripheral Circulation, p. 862. Chapter 30: Assessment and Management of Patients With Vascular Disorders and Problems of Peripheral Circulation - Page 862

A nurse is caring for a client with metastatic breast cancer who is extremely lethargic and very slow to respond to stimuli. The laboratory report indicates a serum calcium level of 12.0 mg/dl, a serum potassium level of 3.9 mEq/L, a serum chloride level of 101 mEq/L, and a serum sodium level of 140 mEq/L. Based on this information, the nurse determines that the client's symptoms are most likely associated with which electrolyte imbalance? Hypocalcemia Hypokalemia Hyperkalemia Hypercalcemia

Hypercalcemia Explanation: The normal reference range for serum calcium is 8.6 to 10.2 mg/dl. A serum calcium level of 12 mg/dl clearly indicates hypercalcemia. The client's other laboratory findings are within their normal ranges, so the client doesn't have hypernatremia, hypochloremia, or hypokalemia. Reference: Hinkle, J. L., Cheever, K. H. Brunner & Suddarth's Textbook of Medical-Surgical Nursing , 14th ed Philadelphia: Wolters Kluwer Health/Lippincott Williams & Wilkins, 2018, Chapter 13: Fluid and Electrolytes: Balance and Disturbance, p. 275. Chapter 13: Fluid and Electrolytes: Balance and Disturbance - Page 275

A 58-year-old with HIV is starting treatment with Combivir. He currently has a CD4+ cell count of less than 200 cells/mL and a viral load greater than 45,000 copies/mL. The nurse treating the client knows that what is a sign of effective drug therapy? Decreased creatinine clearance Decreased CD4+ cell counts Increased CD4+ cell counts Increased red blood cell count

Increased CD4+ cell counts Explanation: Signs of effective drug therapy in clients with HIV include increased CD4+ (T helper) cell counts and decreased viral load. Reference: Chapter 10: Antiviral Agents - Page 157

A female patient comes to the clinic with the complaint that she is having a greenish-colored discharge from the nipple and the breast feels warm to touch. What does the nurse suspect these symptoms may indicate? Cancer A ruptured cyst Infection Blocked lymph duct

Infection Explanation: A green discharge could indicate an infection. Any discharge that is spontaneous, persistent, or unilateral is of concern. Although bloody discharge can indicate a malignancy, it is often caused by a benign wart-like growth on the lining of the duct called an intraductal papilloma. Reference: Hinkle, J.L., and Cheever, K.H. Brunner & Suddarth's Textbook of Medical-Surgical Nursing, 14th ed., Philadelphia: Wolters Kluwer, 2018, Chapter 58: Assessment and Management of Patients With Breast Disorders, Nipple Discharge, p. 1728. Chapter 58: Assessment and Management of Patients With Breast Disorders - Page 1728

A nurse is caring for a patient who is being administered penicillin. What are the common adverse reactions to penicillin a nurse should assess for? Inflammation of the tongue and mouth Sudden loss of consciousness Severe hypotension Impaired oral mucous membranes

Inflammation of the tongue and mouth Explanation: Some of the common adverse effects of penicillin are glossitis (inflammation of the tongue), stomatitis (inflammation of the mouth), and gastritis (inflammation of the stomach). Unless severe, the drug may be continued as prescribed and the nurse would intervene to help the patient manage the common adverse reactions. Impaired oral mucous membranes are signs of a fungal superinfection in the oral cavity, whereas severe hypotension and sudden loss of consciousness are signs of an anaphylactic shock; these are not common adverse effects of penicillin and require immediate medical attention. Reference: Chapter 9: Antibiotics - Page 118

The nurse is formulating a care plan for a client with a seizure disorder. Which intervention would be an appropriate for the nurse to include? Informing the client and family that unrealistic expectations can occur Informing the client and family that excessive frustration is to be expected Informing the client and family that seizure control is gained immediately Informing the client and family that seizure control is not gained immediately

Informing the client and family that seizure control is not gained immediately Explanation: The nurse should inform the client and family that seizure control is not gained immediately when drug therapy is started. The goal is to avoid unrealistic expectations and excessive frustration while drugs and dosages are being changed in an effort to determine the best regimen for the client. Reference: Chapter 23: Anti-seizure Agents - Page 396

A client comes to the walk-in clinic complaining of a "bug in my ear." What action should be taken when there is an insect in the ear? Instillation of mineral oil Use of a small forceps Instillation of carbamide peroxide Instillation of hot water

Instillation of mineral oil Explanation: Mineral oil is instilled into the ear to smother an insect. Carbamide peroxide is used to soften dried cerumen, and small forceps are used to remove solid objects. Hot liquids cause dizziness and should not be instilled in the ear. Reference: Hinkle, J.L., and Cheever, K.H. Brunner & Suddarth's Textbook of Medical-Surgical Nursing, 14th ed., Philadelphia: Wolters Kluwer, 2018, Chapter 64: Assessment and Management of Patients With Hearing and Balance Disorders, Management, p. 1926. Chapter 64: Assessment and Management of Patients With Hearing and Balance Disorders - Page 1926

A nurse is assessing a client with congestive heart failure for jugular vein distension (JVD). Which observation is important to report to the physician? No JVD is present. JVD is noted at the level of the sternal angle. JVD is noted 2 cm above the sternal angle. JVD is noted 4 cm above the sternal angle.

JVD is noted 4 cm above the sternal angle. Explanation: JVD is assessed with the client sitting at a 45° angle. Jugular vein distention greater than 4 cm above the sternal angle is considered abnormal and is indicative of right ventricular failure. Reference: Hinkle, J.L., and Cheever, K.H. Brunner & Suddarth's Textbook of Medical-Surgical Nursing, 14th ed., Philadelphia: Wolters Kluwer, 2018, Chapter 29: Management of Patients With Complications from Heart Disease, p. 828. Chapter 29: Management of Patients With Complications from Heart Disease - Page 828

A client is prescribed doxapram as part of the treatment plan. Which would be most important for the nurse to do when caring for this client? Select all that apply. Avoid giving the client coffee, tea, and carbonated beverages Palpate the bladder for distention Measure urinary intake and output Keep a suction machine nearby Administer drug on an empty stomach

Keep a suction machine nearby Avoid giving the client coffee, tea, and carbonated beverages Measure urinary intake and output Palpate the bladder for distention Explanation: Clients receiving modafinil may develop nausea, so the nurse should keep a suction machine nearby in case the client vomits. Coffee, tea, and carbonated beverages should be avoided when the client is taking a CNS stimulant. Urinary retention may occur, so it is important to monitor the client's intake and output and palpate the bladder for distention. Doxapram is given intravenously. The drug should be administered with food or immediately afterward but not on an empty stomach. Reference: Karch, A.M. Focus on Nursing Pharmacology, 7th ed., Philadelphia: Wolters Kluwer Health, 2017, Chapter 22: Psychotherapeutic Agents, p.

The nurse is performing a physical examination of a patient and observes a well-healed old scar on the right shoulder. The scar is hypertrophied, elevated, and irregular without any redness or irritation. The patient states, "I had shoulder surgery about 5 years ago." The nurse documents this finding as which of the following? Cicatrix Keloid Lichenification Nodule

Keloid Explanation: The hypertrophied, elevated, irregular scar would be documented as a keloid. Lichenification refers to thickening and roughening of the skin or accentuated skin markings that may be due to repeated rubbing, irritation, or scratching. A nodule refers to an elevated, palpable solid mass that extends into the dermis. Cicatrix is another term used to denote a scar. Reference: Hinkle, J.L., and Cheever, K.H. Brunner & Suddarth's Textbook of Medical-Surgical Nursing, 14th ed., Philadelphia: Wolters Kluwer, 2018, Chapter 60: Assessment of Integumentary Function, Table 60-2, p. 1799. Chapter 60: Assessment of Integumentary Function - Page 1799

A client with a traumatic brain injury is showing early signs of increasing intracranial pressure (ICP). While planning care for this client, what would be the priority expected outcome? Attains desired fluid balance Demonstrates optimal cerebral tissue perfusion Maintains a patent airway Displays no signs or symptoms of infection

Maintains a patent airway Explanation: Maintenance of a patent airway is always a first priority. Loss of airway is a possible complication of increasing ICP, as well as aspiration from vomiting. Reference: Hinkle, J.L., and Cheever, K.H. Brunner & Suddarth's Textbook of Medical-Surgical Nursing, 14th ed., Philadelphia: Wolters Kluwer, 2018, Chapter 66: Management of Patients With Neurologic Dysfunction, Maintaining a Patent Airway, p. 1984. Chapter 66: Management of Patients With Neurologic Dysfunction - Page 1984

The triage nurse in the emergency department assesses a 66-year-old male patient who has presented to the emergency department with complaints of midsternal chest pain that has lasted for the last 5 hours. The care team suspects a myocardial infarction (MI). The nurse is aware that, because of the length of time the patient has been experiencing symptoms, the following may have happened to the myocardium: Will probably not have more damage than if he came in immediately May have developed an increased area of infarction Has been damaged already, so immediate treatment is no longer necessary Can have restoration of the area of dead cells with proper treatment

May have developed an increased area of infarction Explanation: When the patient experiences lack of oxygen to myocardium cells during an MI, the sooner treatment is initiated the more likely the treatment will prevent or minimize myocardial tissue necrosis. Despite the length of time the symptoms have been present, treatment needs to be initiated immediately to minimize further damage

The nurse is caring for a client newly diagnosed with sepsis. The client has a serum lactate concentration of 6 mmol/L and fluid resuscitation has been initiated. Which value indicates that the client has received adequate fluid resuscitation? Urine output of 0.2 mL/kg/hr ScvO2 of 60% Mean arterial pressure of 70 mm Hg Central venous pressure of 6 mm Hg

Mean arterial pressure of 70 mm Hg Explanation: The nurse administers fluids to achieve a target central venous pressure of 8 to 12 mm Hg, mean arterial pressure >65 mm Hg, urine output of 0.5 mL/kg/hr, and an ScvO2 of 70%. Reference: Hinkle, J.L., and Cheever, K.H. Brunner & Suddarth's Textbook of Medical-Surgical Nursing, 14th ed., Philadelphia: Wolters Kluwer, 2018, Chapter 14: Shock and Multiple Organ Dysfunction Syndrome, p. 298. Chapter 14: Shock and Multiple Organ Dysfunction Syndrome - Page 298

A nurse expects to find which signs and symptoms in a client experiencing hypoglycemia? Nervousness, diaphoresis, and confusion Polyuria, headache, and fatigue Polyphagia and flushed, dry skin Polydipsia, pallor, and irritability

Nervousness, diaphoresis, and confusion Explanation: Signs and symptoms associated with hypoglycemia include nervousness, diaphoresis, weakness, light-headedness, confusion, paresthesia, irritability, headache, hunger, tachycardia, and changes in speech, hearing, or vision. If untreated, signs and symptoms may progress to unconsciousness, seizures, coma, and death. Polydipsia, polyuria, and polyphagia are symptoms associated with hyperglycemia. Reference: Hinkle, J.L., and Cheever, K.H. Brunner & Suddarth's Textbook of Medical-Surgical Nursing, 14th ed., Philadelphia: Wolters Kluwer, 2018, Chapter 51: Assessment and Management of Patients With Diabetes, Hypoglycemia (Insulin Reactions), p. 1482. Chapter 51: Assessment and Management of Patients With Diabetes - Page 1482

When caring for a client in the third stage of labor, the nurse notices that the expulsion of the placenta has not occurred within 5 minutes after birth of the infant. What should the nurse do? Notify the primary care provider of the problem. Increase the IV tocolytic to help in expulsion of the placenta. Do a vaginal exam to see if the placenta is stuck in the birth canal. Nothing. Normal time for stage three is 5 to 30 minutes.

Nothing. Normal time for stage three is 5 to 30 minutes. Explanation: Following birth, the placenta is spontaneously expelled within 5 to 30 minutes, so there is no problem with this client. No further interventions are needed. Reference: Ricci, S. S., Kyle, T., Carman, S. Maternity and Pediatric Nursing, 3rd ed. Philadelphia: Wolters Kluwer Health, 2017, Chapter 13: Labor and Birth Process, p. 474. Chapter 13: Labor and Birth Process - Page 474

The nurse is assigned a client who had an uneventful colon resection 2 days ago and requires a dressing change. To which nursing team member should the nurse avoid delegating the dressing change? Nursing assistant Registered nurse A senior nursing student present for clinical Licensed practical nurse

Nursing assistant Explanation: The nurse should avoid delegating the dressing change to the nursing assistant. The dressing change would be within the scope of practice of the registered nurse, licensed practical nurse, and the senior nursing student. Reference: Chapter 17: Implementing - Page 429

The nurse is coordinating care for a client with continuous pulse oximetry who requires pharyngeal suctioning. To which staff member should the nurse avoid delegating the task of suctioning? Nursing assistant who is a nursing student Licensed practical nurse Registered nurse A senior nursing student present for clinical

Nursing assistant who is a nursing student Explanation: The nurse should avoid delegating this client to the nursing assistant who is a nursing student. Suctioning and the associated evaluation of the client would be within the scope of practice of the registered nurse, licensed practical nurse, and the senior nursing student present for clinical. Reference:

A client presents to the emergency room with abdominal pain and upper gastrointestinal bleeding. The client is sweating and appears to be in moderate distress. Which nursing action would be a priority at this time? Assess bowel sounds and abdominal tenderness. Insert an NG tube and connect to suction. Document history of the symptoms. Obtain vital signs.

Obtain vital signs. Explanation: The priority nursing action is vital signs. Vital signs provide valuable information on the internal body system. Symptoms of shock, such as low blood pressure, a rapid weak pulse, cold clammy skin, and restlessness, can be monitored. Assessing bowel sounds and abdominal tenderness can provide useful data but is not a priority. Documentation is a lower priority and a health care provider's order is needed for a nasogastric tube placement.

A client is diagnosed with peptic ulcer due to H. pylori. The nurse would anticipate administering which agent in conjunction with an antibiotic? Omeprazole Magaldrate Sucralfate Cimetidine

Omeprazole Explanation: Omeprazole is a proton pump inhibitor which is used as part of combination therapy to treat H. pylori infections. Magaldrate is an antacid that is used to relieve GI hyperacidity. Sucralfate is a GI protectant that is used as short-term treatment of duodenal ulcers. Cimetidine is a H-2 receptor antagonist used to treat duodenal and benign gastric ulcers. Reference: Chapter 57: Drugs Affecting Gastrointestinal Secretions - Page 1007

The nurse is caring for a client with intussusception of the bowel. What does the nurse understand occurs with this disorder? A loop of intestine adheres to an area that is healing slowly after surgery. The bowel twists and turns itself and obstructs the intestinal lumen. One part of the intestine telescopes into another portion of the intestine. The bowel protrudes through a weakened area in the abdominal wall.

One part of the intestine telescopes into another portion of the intestine. Explanation: In intussusception of the bowel, one part of the intestine telescopes into another portion of the intestine. When the bowel twists and turns itself and obstructs the intestinal lumen, this is known as a volvulus. A hernia is when the bowel protrudes through a weakened area in the abdominal wall. An adhesion is a loop of intestine that adheres to an area that is healing slowly after surgery. Reference: Hinkle, J.L., and Cheever, K.H. Brunner & Suddarth's Textbook of Medical-Surgical Nursing, 14th ed., Philadelphia: Wolters Kluwer, 2018, Chapter 47: Management of Patients With Intestinal and Rectal Disorders, Table 47-4: Mechanical Causes of Intestinal Obstruction, p. 1328. Chapter 47: Management of Patients With Intestinal and Rectal Disorders - Page 1328

The nurse is admitting a patient with COPD. The decrease of what substance in the blood gas analysis would indicate to the nurse that the patient is experiencing hypoxemia? pH PaO2 PCO2 HCO3

PaO2 Explanation: Hypoxemic hypoxia, or hypoxemia, is a decreased oxygen level in the blood (PaO2) resulting in decreased oxygen diffusion into the tissues. Reference: Hinkle, J. L., Cheever, K. H. Brunner & Suddarth's Textbook of Medical-Surgical Nursing , 14th ed Philadelphia: Wolters Kluwer Health/Lippincott Williams & Wilkins, 2018, Chapter 21: Respiratory Care Modalities, p. 511. Chapter 21: Respiratory Care Modalities - Page 511

The anesthesiologist is administering a stable and safe nondepolarizing muscle relaxant. What medication does the nurse anticipate will be administered? Norcuron (vecuronium bromide) Anectine (succinylcholine chloride) Pavulon (pancuronium bromide) Syncurine (decamethonium)

Pavulon (pancuronium bromide) Explanation: Pancuronium (Pavulon) is a nondepolarizing muscle relaxant with a longer onset and duration. Succinylcholine (Anectine) and decamethonium (Syncurine) are depolarizing muscle relaxants. Vercuronium (Norcuron) is a nondepolarizing muscle relaxant that requires mixing. Reference: Hinkle, J.L., and Cheever, K.H. Brunner & Suddarth's Textbook of Medical-Surgical Nursing, 14th ed., Philadelphia: Wolters Kluwer, 2018, Chapter 18: Intraoperative Nursing Management, Table 18-2, p. 447. Chapter 18: Intraoperative Nursing Management - Page 447

The nurse is preparing an outline for a class on the physiology of the male sexual response. Which event would the nurse identify as occurring first? Ejaculation Psychological release Sperm emission Penile vasodilation

Penile vasodilation Explanation: With sexual stimulation, the arteries leading to the penis dilate and increase blood flow into erectile tissue. Blood accumulates, causing the penis to swell and elongate. Sperm emission (movement of sperm from the testes and fluid from the accessory glands) occurs with orgasm. Orgasm results in a pleasurable feeling of physiologic and psychological release. Ejaculation results in the discharge of semen from the urethra. Reference: Chapter 39: Introduction to the Reproductive System - Page 668-669

Which is a symptom of severe thrombocytopenia? Dyspnea Inflammation of the tongue Inflammation of the mouth Petechiae

Petechiae Explanation: Clients with severe thrombocytopenia have petechiae, which are pinpoint hemorrhagic lesions, usually more prominent on the trunk or anterior aspects of the lower extremities. Reference: Hinkle, J.L., and Cheever, K.H. Brunner & Suddarth's Textbook of Medical-Surgical Nursing, 14th ed., Philadelphia: Wolters Kluwer, 2018, Chapter 32: Assessment of Hematologic Function and Treatment Modalities, Table 32-2, p. 911. Chapter 32: Assessment of Hematologic Function and Treatment Modalities - Page 911

The nurse is caring for a client that has undergone a colon resection. While turning the client, wound dehiscence with evisceration occurs. What is the nurse's first response? Return the client to their back. Call the health care provider. Take a blood pressure and pulse. Place saline-soaked sterile dressings on the wound.

Place saline-soaked sterile dressings on the wound. Explanation: The nurse should first place saline-soaked sterile dressings on the open wound to prevent tissue drying and possible infection. Then the nurse should call the health care provider and take the client's vital signs. The dehiscence needs to be surgically closed, so the nurse should never try to close it.

A client with acquired immune deficiency syndrome (AIDS) is exhibiting shortness of breath, cough, and fever. What type of infection will the nurse most likely suspect? Cytomegalovirus Pneumocystis jiroveci Legionella Mycobacterium avium complex

Pneumocystis jiroveci Explanation: Although mycobacterium, legionella, and cytomegalovirus may cause the signs and symptoms described, the most common infection in people with AIDS is pneumocystis pneumonia caused by pneumocystis jiroveci. It is the most common opportunistic infection associated with AIDS. Reference: Hinkle, J.L., and Cheever, K.H. Brunner & Suddarth's Textbook of Medical-Surgical Nursing, 14th ed., Philadelphia: Wolters Kluwer, 2018, Chapter 36: Management of Patients With Immune Deficiency Disorders, Respiratory Manifestations, p. 1037. Chapter 36: Management of Patients With Immune Deficiency Disorders - Page 1037

Which treatment should take place immediately in a client experiencing autonomic dysreflexia? Increase the client's blood pressure to allow for adequate perfusion. Apply binders and support hose to shunt the blood to the main organs. Place the client in a supine position, and increase intravenous fluids. Position the client in upright position, and correct the initiating stimulus.

Position the client in upright position, and correct the initiating stimulus. Explanation: Autonomic dysreflexia is a clinical emergency, and requires monitoring of blood pressure while correcting the initiating stimulus (e.g., full bladder, pain). The nurse should place the client in an upright position and remove all support hose binders to promote venous pooling to help decrease an extremely elevated blood pressure. Intravenous fluids are not an immediate intervention for this condition. Reference: Chapter 15: Disorders of Motor Function - Page 407

The nurse is reviewing the medical record of a client with glaucoma. Which of the following would alert the nurse to suspect that the client was at increased risk for this disorder? Age younger than 40 years Prolonged use of corticosteroids Hyperopia since age 20 years History of respiratory disease

Prolonged use of corticosteroids Explanation: Risk factors associated with glaucoma include prolonged use of topical or systemic corticosteroids, older age, myopia, and a history of cardiovascular disease. Reference: Hinkle, J.L., and Cheever, K.H. Brunner & Suddarth's Textbook of Medical-Surgical Nursing, 14th ed., Philadelphia: Wolters Kluwer, 2018, Chapter 63: Assessment and Management of Patients With Eye and Vision Disorders, Corticosteroids and Nonsteroidal Anti-Inflammatory Drugs, p. 1888. Chapter 63: Assessment and Management of Patients With Eye and Vision Disorders - Page 1888

A patient sustained a head injury and has been admitted to the neurosurgical intensive care unit (ICU). The patient began having seizures and was administered a sedative-hypnotic medication that is ultra-short acting and can be titrated to patient response. What medication will the nurse be monitoring during this time? Phenobarbital Lorazepam (Ativan) Midazolam (Versed) Propofol (Diprivan)

Propofol (Diprivan) Explanation: If the patient is very agitated, benzodiazepines are the most commonly used sedative agents and do not affect cerebral blood flow or ICP. Lorazepam (Ativan) and midazolam (Versed) are frequently used but have active metabolites that my cause prolonged sedation, making it difficult to conduct a neurologic assessment. Propofol ( Diprivan), on the other hand, a sedative-hypnotic agent that is supplied in an intralipid emulsion for intravenous (IV) use, is the sedative of choice. It is an ultra-short acting, rapid onset drug with elimination half-life of less than an hour. It has a major advantage of being titratable to its desired clinical effect but still provides the opportunity for an accurate neurologic assessment (Hickey, 2009). Reference: Hinkle, J.L., and Cheever, K.H. Brunner & Suddarth's Textbook of Medical-Surgical Nursing, 14th ed., Philadelphia: Wolters Kluwer, 2018, Chapter 68: Management of Patients With Neurologic Trauma, Supportive Measures, p. 2038. Chapter 68: Management of Patients With Neurologic Trauma - Page 2038 Add a Note

A client exhibits signs and symptoms of heparin overdose. The nurse would anticipate administering: Vitamin K Drotrecogin alfa Protamine sulfate Urokinase

Protamine sulfate Explanation: Protamine sulfate is the antidote for heparin overdose. Vitamin K is the antidote for warfarin overdose. Urokinase is a thrombolytic. Drotrecogin alfa is a C-reactive protein that has anticoagulant effects. Reference: Chapter 48: Drugs Affecting Blood Coagulation - Page 846

A client is admitted to the hospital with systolic left-sided heart failure. The nurse knows to look for which assessment finding for this client? Nausea Jugular venous distention Pedal edema Pulmonary congestion

Pulmonary congestion Explanation: When the left ventricle cannot effectively pump blood out of the ventricle into the aorta, the blood backs up into the pulmonary system and causes congestion, dyspnea, and shortness of breath. All the other choices are symptoms of right-sided heart failure. They are all symptoms of systolic failure. Reference: Hinkle, J.L., and Cheever, K.H. Brunner & Suddarth's Textbook of Medical-Surgical Nursing, 14th ed., Philadelphia: Wolters Kluwer, 2018, Chapter 29: Management of Patients With Complications from Heart Disease, Left-Sided Heart Failure, p. 822. Chapter 29: Management of Patients With Complications from Heart Disease - Page 822

Which of the following would least suggest emotional health? Acceptance of reality Realistic sense of hopelessness Pursuit of personal goals Management of every day challenges

Realistic sense of hopelessness Explanation: Emotional health is manifested by maintaining a realistic sense of hope as well as by the achievement of personal goals, ability to manage every day challenges and problems, and an acceptance of reality. Reference: Hinkle, J. L., Cheever, K. H. Brunner & Suddarth's Textbook of Medical-Surgical Nursing , 14th ed Philadelphia: Wolters Kluwer Health/Lippincott Williams & Wilkins, 2018, Chapter 6: Individual and Family Homeostasis, Stress, and Adaptation, p. 87. Chapter 6: Individual and Family Homeostasis, Stress, and Adaptation - Page 87

The nurse observes that a postsurgical client has hemorrhaged and is in hypovolemic shock. Which nursing intervention will manage and minimize hemorrhage and shock? Reinforcing the dressing or applying pressure if bleeding is frank Elevating the head of the bed Monitoring vital signs every 15 minutes Encouraging the client to breathe deeply

Reinforcing the dressing or applying pressure if bleeding is frank Explanation: The nurse should reinforce the dressing or apply pressure if bleeding is frank. The nurse should keep the head of the bed flat unless it is contraindicated. Encouraging the client to breathe deeply will not help manage and minimize hemorrhage and shock. Monitoring vital signs every 15 minutes is an appropriate nursing intervention but will not minimize hemorrhage and shock; it will just help to determine the extent and progression of the problem. Reference: Hinkle, J.L., and Cheever, K.H. Brunner & Suddarth's Textbook of Medical-Surgical Nursing, 14th ed., Philadelphia: Wolters Kluwer, 2018, Chapter 19: Postoperative Nursing Management, p. 459. Chapter 19: Postoperative Nursing Management - Page 459

A nurse is aware that after a burn injury and respiratory difficulties have been managed, the next most urgent need is to: Monitor cardiac status. Replace lost fluids and electrolytes. Prevent renal shutdown. Measure hourly urinary output.

Replace lost fluids and electrolytes. Explanation: After managing respiratory difficulties, the next most urgent need is to prevent irreversible shock by replacing lost fluids and electrolytes. The total volume and rate of IV fluid replacement are gauged by the patient's response and guided by the resuscitation formula. Reference: Hinkle, J.L., and Cheever, K.H. Brunner & Suddarth's Textbook of Medical-Surgical Nursing, 14th ed., Philadelphia: Wolters Kluwer, 2018, Chapter 62: Management of Patients with Burn Injury, Fluid and Electrolyte Alterations, p. 1851. Chapter 62: Management of Patients with Burn Injury - Page 1851

When assessing an older adult, the nurse anticipates an increase in which component of respiratory status? Cough efficiency Residual lung volume Vital capacity Gas exchange and diffusing capacity

Residual lung volume Explanation: With an increase in residual lung volume the client experiences fatigue and breathlessness with sustained activity. The nurse anticipates decreased vital capacity. The nurse anticipates decreased gas exchange and diffusing capacity resulting in impaired healing of tissues due to decreased oxygenation. The nurse anticipates difficulty coughing up secretions due to decreased cough efficiency. Reference: Hinkle, J.L., and Cheever, K.H. Brunner & Suddarth's Textbook of Medical-Surgical Nursing, 14th ed., Philadelphia: Wolters Kluwer, 2018, Chapter 11: Health Care of the Older Adult, Table 11-1, p. 197. Chapter 11: Health Care of the Older Adult - Page 197

The nurse is caring for a client exposed to a blistering agent. While the nurse is quickly decontaminating the client by showering and bagging all client clothing, what is the nurse simultaneously assessing for? Respiratory compromise Neurological compromise Sensory neglect Cardiovascular compromise

Respiratory compromise Explanation: A person exposed to a blistering agent or vesicant must be decontaminated immediately, with clothing removed and bagged. Irrigation of the victim's eyes and application of topical analgesia, antibiotics, and lubricants to the skin occur. Simultaneously, the nurse is assessing the respiratory system for airway obstruction because blisters from inhaled toxics can swell obstructing respiratory passages. Reference: Hinkle, J.L., and Cheever, K.H. Brunner & Suddarth's Textbook of Medical-Surgical Nursing, 14th ed., Philadelphia: Wolters Kluwer, 2018, Chapter 73: Terrorism, Mass Casualty, and Disaster Nursing, Clinical Manifestations, p. 2207. Chapter 73: Terrorism, Mass Casualty, and Disaster Nursing - Page 2207

Nursing students are reviewing the pathophysiology of human immunodeficiency virus (HIV). They demonstrate understanding of the information when they state which of the following as the form of the genetic viral material? Viral core Deoxyribonucleic acid (DNA) Glycoprotein envelope Ribonucleic acid (RNA)

Ribonucleic acid (RNA) Explanation: HIV is a retrovirus that carries its genetic material in the form of RNA rather than DNA. HIV consists of a viral core containing the viral RNA, surrounded by an envelope consisting of protruding glycoproteins. Reference: Hinkle, J.L., and Cheever, K.H. Brunner & Suddarth's Textbook of Medical-Surgical Nursing, 14th ed., Philadelphia: Wolters Kluwer, 2018, Chapter 36: Management of Patients With Immune Deficiency Disorders, Pathophysiology, p. 1030. Chapter 36: Management of Patients With Immune Deficiency Disorders - Page 1030

A client is admitted with a diagnosis of acute appendicitis. When assessing the abdomen, the nurse would expect to find rebound tenderness at which location? Left upper quadrant Right lower quadrant Right upper quadrant Left lower quadrant

Right lower quadrant Explanation: Reference: Hinkle, J.L., and Cheever, K.H. Brunner & Suddarth's Textbook of Medical-Surgical Nursing, 14th ed., Philadelphia: Wolters Kluwer, 2018, Chapter 47: Management of Patients With Intestinal and Rectal Disorders, Clinical Manifestations, p. 1323. Chapter 47: Management of Patients With Intestinal and Rectal Disorders - Page 1323

The nurse observes a client during an exercise stress test (bicycle). Which finding indicates a positive test and the need for further diagnostic testing? Heart rate changes; 78 bpm to 112 bpm Dizziness and leg cramping BP changes; 148/80 mm Hg to 166/90 mm Hg ST-segment changes on the ECG

ST-segment changes on the ECG Explanation: During the test, the following are monitored: two or more ECG leads for heart rate, rhythm, and ischemic changes; blood pressure; skin temperature; physical appearance; perceived exertion; and symptoms, including chest pain, dyspnea, dizziness, leg cramping, and fatigue. The test is terminated when the target heart rate is achieved or if the client experiences signs of myocardial ischemia. Further diagnostic testing, such as a cardiac catheterization, may be warranted if the client develops chest pain, extreme fatigue, a decrease in blood pressure or pulse rate, serious dysrhythmias or ST-segment changes on the ECG during the stress test. The other findings would not warrant stopping the test. Reference: Hinkle, J.L., and Cheever, K.H. Brunner & Suddarth's Textbook of Medical-Surgical Nursing, 14th ed., Philadelphia: Wolters Kluwer, 2018, Chapter 25: Assessment of Cardiovascular Function, p. 699. Chapter 25: Assessment of Cardiovascular Function - Page 699

In which stage is a pressure ulcer considered a partial-thickness wound? Stage IV Stage III Stage I Stage II

Stage II Explanation: A stage II pressure ulcer is considered a partial-thickness wound. A stage I pressure ulcer is an area of erythema that does not blanch with pressure. A stage III pressure ulcer extends into the subcutaneous tissue. A stage IV pressure ulcer extends to the underlying muscle and bone. Reference: Hinkle, J.L., and Cheever, K.H. Brunner & Suddarth's Textbook of Medical-Surgical Nursing, 14th ed., Philadelphia: Wolters Kluwer, 2018, Chapter 10: Principles and Practices of Rehabilitation, Chart 10-7, p. 182. Chapter 10: Principles and Practices of Rehabilitation - Page 182

A nurse is in the cafeteria at work. A fellow worker at another table suddenly stands up, leans forward with hands crossed at the neck, and makes gasping noises. The nurse first Exerts pressure against the worker's abdomen Places both arms around the worker's waist Stands behind the worker, who has hands across the neck Makes a fist with one hand with the thumb outside the fist

Stands behind the worker, who has hands across the neck Explanation: The description of the fellow worker is a person who is choking. Following guidelines set by the American Heart Association, the nurse first stands behind the person who is choking. Reference: Hinkle, J. L., Cheever, K. H. Brunner & Suddarth's Textbook of Medical-Surgical Nursing , 14th ed Philadelphia: Wolters Kluwer Health/Lippincott Williams & Wilkins, 2018, Chapter 22: Management of Patients With Upper Respiratory Tract Disorders, p. 579. Chapter 22: Management of Patients With Upper Respiratory Tract Disorders - Page 579

A patient is prescribed daily doses of phenytoin for seizures. The nurse knows that a single dose should not be missed during the course of treatment. Which condition could result if a dose is missed? CNS depression Status epilepticus Hypotension Nystagmus

Status epilepticus Explanation: Status epilepticus may result from abrupt discontinuation of the drug, even when the anticonvulsant is being administered in small daily doses. Abrupt discontinuation of the drug does not cause CNS depression, hypotension, or nystagmus. CNS depression, hypotension, and nystagmus are adverse reactions of phenytoin. Reference: Chapter 23: Anti-seizure Agents - Page 396

While administering a cleansing enema, the client displays lightheadedness, nausea, and has clammy skin. The nurse would implement which priority action? Slow the infusion rate, have the client take deep breaths, then resume the enema. Slow the infusion rate, withdraw the tubing slightly, then resume the enema. Stop the procedure, monitor heart rate and blood pressure. Stop the procedure and reposition the client.

Stop the procedure, monitor heart rate and blood pressure. Explanation: When administering an enema, the client's vagus nerve may be stimulated, causing a decrease in the heart rate. The client will exhibit nausea, lightheadedness, dizziness, and clammy skin. The procedure should be stopped, heart rate and blood pressure monitored, and the health care provider notified. The other responses are not appropriate for a client exhibiting a vagal response. Reference: Chapter 38: Bowel Elimination - Page 1439

What is the action of codeine when used to treat a cough? Expectorant Antisuppressant Suppressant Antihistamine

Suppressant Explanation: Codeine, which is an ingredient in many cough preparations, is generally considered to be the preferred cough suppressant ingredient. Reference: Chapter 39: Oxygenation and Perfusion - Page 1505

Morton neuroma is exhibited by which clinical manifestation? Longitudinal arch of the foot is diminished Swelling of the third (lateral) branch of the median plantar nerve Inflammation of the foot-supporting fascia High arm and a fixed equinus deformity

Swelling of the third (lateral) branch of the median plantar nerve Explanation: Morton neuroma is swelling of the third branch of the median plantar nerve. Pes cavus refers to a foot with an abnormally high arch and a fixed equinus deformity of the forefoot. Flatfoot is a common disorder in which the longitudinal arch of the foot is diminished. Plantar fasciitis is an inflammation of the foot-supporting fascia. Reference: Hinkle, J.L., and Cheever, K.H. Brunner & Suddarth's Textbook of Medical-Surgical Nursing, 14th ed., Philadelphia: Wolters Kluwer, 2018, Chapter 41: Management of Patients With Musculoskeletal Disorders, p. 1168. Chapter 41: Management of Patients With Musculoskeletal Disorders - Page 1168

A client is being sent home with subcutaneous heparin after a total hip replacement. The nurse understands what symptom would indicate a serious drug reaction? Headache Hypotension Tarry stools Stomach pain

Tarry stools Explanation: Tarry stools would be an indication of gastrointestinal bleeds. The most common adverse effect of heparin is bleeding. Reference: Chapter 48: Drugs Affecting Blood Coagulation - Page 845

What should the nurse do to manage the persistent swelling in a client with severe lymphangitis and lymphadenitis? Avoid elevating the area Offer cold applications to promote comfort and to enhance circulation Inform the physician if the client's temperature remains low Teach the client how to apply an elastic sleeve

Teach the client how to apply an elastic sleeve Explanation: In severe cases of lymphangitis and lymphadenitis with persistent swelling, the nurse teaches the client how to apply an elastic sleeve or stocking. The nurse informs the physician if the client's temperature remains elevated. The nurse recommends elevating the area to reduce the swelling and provides warmth to promote comfort and to enhance circulation. Reference: Hinkle, J.L., and Cheever, K.H. Brunner & Suddarth's Textbook of Medical-Surgical Nursing, 14th ed., Philadelphia: Wolters Kluwer, 2018, Chapter 30: Assessment and Management of Patients With Vascular Disorders and Problems of Peripheral Circulation, p. 880. Chapter 30: Assessment and Management of Patients With Vascular Disorders and Problems of Peripheral Circulation - Page 880

A client is experiencing anorexia, myalgia, arthralgia, headache, and fatigue. The nurse should assess for: Respirations Hypothermia Temperature Urinary output

Temperature Explanation: Common clinical manifestations of fever include anorexia, myalgia, arthralgia, headaches, and fatigue; thus, the nurse should assess the client's temperature. Reference: Chapter 9: Inflammation, Tissue Repair, and Wound Healing - Page 371 Add a Note

The nurse is caring for a client during an intra operative procedure. When assessing vital signs, which result indicates a need to alert the anesthesiologist immediately? Blood pressure of 104/62 mm Hg Respiratory rate of 18 breaths/min Pulse rate of 110 beats/min Temperature of 102.5°F (39°C)

Temperature of 102.5°F (39°C) Explanation: Intra operative hyperthermia can indicate a life-threatening condition called malignant hyperthermia. The circulating nurse closely monitors the client for signs of hyperthermia. The pulse rate, respiratory rate, and blood pressure did not indicate a significant concern. Reference: Hinkle, J.L., and Cheever, K.H. Brunner & Suddarth's Textbook of Medical-Surgical Nursing, 14th ed., Philadelphia: Wolters Kluwer, 2018, Chapter 18: Intraoperative Nursing Management, Malignant Hyperthermia, p. 450. Chapter 18: Intraoperative Nursing Management - Page 450

A client has been treated for migraine headaches for several months and comes to the clinic stating he is getting no better. The nurse is talking with the client and hears an audible click when the client is moving his jaw. What does the nurse suspect may be happening? Trigeminal neuralgia Dislocated jaw Temporomandibular disorder Loose teeth

Temporomandibular disorder Explanation: The disorder can be confused with trigeminal neuralgia and migraine headaches. The client experiences clicking of the jaw when moving the joint, or the jaw can lock, which interferes with opening the mouth. Loose teeth will not cause a clicking of the jaw. The client does not have a dislocated jaw. Reference: Hinkle, J.L., and Cheever, K.H. Brunner & Suddarth's Textbook of Medical-Surgical Nursing, 14th ed., Philadelphia: Wolters Kluwer, 2018, Chapter 41: Management of Patients With Musculoskeletal Disorders, Clinical Manifestations, p. 1269. Chapter 41: Management of Patients With Musculoskeletal Disorders - Page 1269

The nurse received the report from a previous shift. One of her clients was reported to have a history of basilar skull fracture with otorrhea. What assessment finding does the nurse anticipate? The client has cerebral spinal fluid (CSF) leaking from the ear. The client has serous drainage from the nose. The client has an elevated temperature. The client has ecchymosis in the periorbital region.

The client has cerebral spinal fluid (CSF) leaking from the ear. Explanation: Otorrhea means leakage of CSF from the ear. The client with a basilar skull fracture can create a pathway from the brain to the middle ear due to a tear in the dura. As a result, the client can have cerebral spinal fluid leak from the ear. The nurse may assess clear fluid in the ear canal. Ecchymosis and periorbital edema can be present as a manifestation of bruising from the head injury. An elevated temperature may occur from the head injury and is monitored closely. The client may have serous drainage from the nose especially immediately following the injury. Reference: Hinkle, J.L., and Cheever, K.H. Brunner & Suddarth's Textbook of Medical-Surgical Nursing, 14th ed., Philadelphia: Wolters Kluwer, 2018, Chapter 68: Management of Patients With Neurologic Trauma, Clinical Manifestations, p. 2035. Chapter 68: Management of Patients With Neurologic Trauma - Page 2035

The nurse is obtaining physician orders which include a pulse pressure. The nurse is most correct to report which of the following? The difference between the arterial and venous blood pressure The difference between an upper extremity and lower extremity blood pressure The difference between an apical and radial pulse The difference between the systolic and diastolic pressure

The difference between the systolic and diastolic pressure Explanation: The nurse would report the difference between the systolic blood pressure number and the diastolic blood pressure number as the pulse pressure. Reference: Hinkle, J.L., and Cheever, K.H. Brunner & Suddarth's Textbook of Medical-Surgical Nursing, 14th ed., Philadelphia: Wolters Kluwer, 2018, Chapter 14: Shock and Multiple Organ Dysfunction Syndrome, Monitoring Tissue Perfusion, p. 300. Chapter 14: Shock and Multiple Organ Dysfunction Syndrome - Page 300 Add a Note

The patient has a heart rate of 72 bpm with a regular rhythm. Where does the nurse determine the impulse arises from? The Purkinje fibers The sinoatrial node The AV node The ventricles

The sinoatrial node Explanation: The sinoatrial node, the primary pacemaker of the heart, in a normal resting adult heart has an inherent firing rate of 60 to 100 impulses per minute; however, the rate changes in response to the metabolic demands of the body (Weber & Kelley, 2010). Reference: Hinkle, J.L., and Cheever, K.H. Brunner & Suddarth's Textbook of Medical-Surgical Nursing, 14th ed., Philadelphia: Wolters Kluwer, 2018, Chapter 25: Assessment of Cardiovascular Function, Cardiac Electrophysiology, p. 675. Chapter 25: Assessment of Cardiovascular Function - Page 675

A client is to have an upper GI procedure with barium ingestion and abdominal ultrasonography. While scheduling these diagnostic tests, the nurse must consider which factor? The ultrasonography should be scheduled before the GI procedure. Both tests need to be done before breakfast. The client may eat a light meal before either test. The upper GI should be scheduled before the ultrasonography.

The ultrasonography should be scheduled before the GI procedure. Explanation: Both an upper GI procedure with barium ingestion and an ultrasonography may be completed on the same day. The ultrasonography test should be completed first, because the barium solution could interfere with the transmission of the sound waves. The ultrasonography test uses sound waves that are passed into internal body structures, and the echoes are recorded as they strike tissues. Fluid in the abdomen prevents transmission of ultrasound. Reference: Hinkle, J.L., and Cheever, K.H. Brunner & Suddarth's Textbook of Medical-Surgical Nursing, 14th ed., Philadelphia: Wolters Kluwer, 2018, Chapter 43: Assessment of Digestive and Gastrointestinal Function, Abdominal Ultrasonography, p. 1233. Chapter 43: Assessment of Digestive and Gastrointestinal Function - Page 1233

During a pharmacology class, the students are told that some drugs need to be closely monitored. What aspect should the nurse closely monitor for in clients who have been administered salicylates, loop diuretics, quinidine, quinine, or aminoglycosides? Signs of hypotension Reduced urinary output Tinnitus and sensorineural hearing loss Impaired facial movement

Tinnitus and sensorineural hearing loss Explanation: Reference: Hinkle, J.L., and Cheever, K.H. Brunner & Suddarth's Textbook of Medical-Surgical Nursing, 14th ed., Philadelphia: Wolters Kluwer, 2018, Chapter 64: Assessment and Management of Patients With Hearing and Balance Disorders, Ototoxicity, p. 1937. Chapter 64: Assessment and Management of Patients With Hearing and Balance Disorders - Page 1937

A client comes to the clinic for a follow-up visit. During the interview, the client states, "Sometimes when I have to urinate I can't control it, and do not reach the bathroom in time." The nurse suspects that the client is experiencing which type of incontinence? Functional Urge Stress Overflow

Urge Explanation: Urge incontinence occurs when the client experiences the sensation to void but cannot control voiding in time to reach a toilet. Stress incontinence occurs when the client has an involuntary loss of urine that results from a sudden increase in intra-abdominal pressure. Overflow incontinence occurs when the client experiences an involuntary loss of urine related to an overdistended bladder; the client voids small amounts frequently and dribbles. Functional incontinence occurs when the client has function of the lower urinary tract but cannot identify the need to void or ambulate to the toilet. Reference: Hinkle, J.L., and Cheever, K.H. Brunner & Suddarth's Textbook of Medical-Surgical Nursing, 14th ed., Philadelphia: Wolters Kluwer, 2018, Chapter 55: Management of Patients With Urinary Disorders, Types of Urinary Incontinence, p. 1624. Chapter 55: Management of Patients With Urinary Disorders - Page 1624

A client comes to the emergency department complaining of a sudden onset of sharp, severe flank pain. During the physical examination, the client indicates that the pain, which comes in waves, travels to the suprapubic region. He states, "I can even feel the pain at the tip of my penis." Which of the following would the nurse suspect? Renal cell carcinoma Acute glomerulonephritis Ureteral stricture Urinary calculi

Urinary calculi Explanation: Symptoms of a kidney or ureteral stone vary with size, location, and cause. Small stones may pass unnoticed; however, sudden, sharp, severe flank pain that travels to the suprapubic region and external genitalia is the classic symptom of urinary calculi. The pain is accompanied by renal or ureteral colic, painful spasms that attempt to move the stone. The pain comes in waves that radiate to the inguinal ring, the inner aspect of the thigh, and to the testicle or tip of the penis in men, or the urinary meatus or labia in women. Clients with acute glomerulonephritis may be asymptomatic or may exhibit fever, nausea, malaise, headache, edema (generalized or periorbital), pain, and mild to moderate hypertension. Clients with ureteral stricture may complain of flank pain and tenderness at the costovertebral angle and back or abdominal discomfort. A client with renal cell carcinoma rarely exhibits symptoms early on but may present with painless hematuria and persistent back pain in later stages. Reference: Hinkle, J.L., and Cheever, K.H. Brunner & Suddarth's Textbook of Medical-Surgical Nursing, 14th ed., Philadelphia: Wolters Kluwer, 2018, Chapter 55: Management of Patients With Urinary Disorders, Clinical Manifestations, p. 1574. Chapter 55: Management of Patients With Urinary Disorders - Page 1574

The nurse is teaching a client about cancer prevention. The nurse evaluates teaching as most effective when a female client states that she will Obtain a cancer history from her parents. Decrease tobacco smoking from one pack/day to half a pack/day. Exercise 30 minutes 3 times each week. Use sunscreen when outdoors.

Use sunscreen when outdoors. Explanation: Use of sunscreens play a role in the amount of exposure to ultraviolet light. Even decreasing the use of tobacco still exposes a person to risk of cancer. The American Cancer Society recommends adults to engage in at least 30 minutes of moderate to vigorous physical activity on 5 or more days each week. It is recommended to obtain a cancer history from at least three generations. Reference: Hinkle, J.L., and Cheever, K.H. Brunner & Suddarth's Textbook of Medical-Surgical Nursing, 14th ed., Philadelphia: Wolters Kluwer, 2018, Chapter 15: Management of Patients with Oncologic Disorders, Physical Agents, p. 328. Chapter 15: Management of Patients with Oncologic Disorders - Page 328 Add a Note

A nurse is preparing to measure a client's urine output. Which interventions would be of highest priority? Measuring the urine container at eye level Wearing gloves when handling the urine Noting the color and clarity of the urine Using an appropriate measuring container

Wearing gloves when handling the urine Explanation: All of these interventions would be important to ensure safety in handling the client's urine and obtaining an accurate output. However, safety with handling body fluids would be a priority for the nurse to decrease risk of exposure to pathogens or blood that may be in the client's urine. Reference: Chapter 37: Urinary Elimination - Page 1349

The nurse is reviewing laboratory values on a client with heart failure and atrial fibrillation. The client has a potassium level of 2.8 mEq/L (2.8 mmol/L). The client is scheduled to receive their 0900 dose of digoxin. What is the nurse's best action? Review the dietary needs of the client and consult the dietitian. Draw a stat potassium level and compare the earlier result with the current result. Withhold the dose of digoxin and notify the healthcare provider. Give half of the digoxin and offer potassium-rich foods all day. Administer the dose of digoxin and offer the client a banana with breakfast.

Withhold the dose of digoxin and notify the healthcare provider. Explanation: Administering the dose of digoxin should not be done. The effect of digoxin is enhanced in the presence of hypokalemia and digoxin toxicity may occur. Drawing a stat potassium level and comparing the earlier result with the current result does not address the low potassium level. The level could be lower, putting the client at risk for a cardiac event. Offering the client a banana with breakfast will not raise the potassium level because the banana does not contain enough potassium. Withholding the dose of digoxin should be done to prevent digoxin toxicity. The nurse should notify the healthcare provider to let them know about the low potassium level. The healthcare provider can order a potassium supplement orally or intravenously and another potassium level laboratory value to be drawn after treatment to evaluate if the level is within normal limits after treatment. Giving half of the digoxin may cause digoxin toxicity because of the low potassium level. Offering potassium-rich foods all day will not do much to increase the potassium level. Reviewing the dietary needs of the client and consulting the dietitian is not warranted at this time.

Which are risk factors for spinal cord injury (SCI)? Select all that apply. Young age Female gender Drug abuse Alcohol use European American ethnicity

Young age Alcohol use Drug abuse Explanation: The predominant risk factors for SCI include young age, male gender, and alcohol and drug use. The frequency with which these risk factors are associated with SCI emphasizes the importance of primary prevention. Reference: Hinkle, J.L., and Cheever, K.H. Brunner & Suddarth's Textbook of Medical-Surgical Nursing, 14th ed., Philadelphia: Wolters Kluwer, 2018, Chapter 68: Management of Patients With Neurologic Trauma, p. 2048. Chapter 68: Management of Patients With Neurologic Trauma - Page 2048

To which client would the nurse be most likely to administer a PRN medication? a client who requires daily medication to control hypertension a client who is reporting pain near the surgical site a client who is experiencing severe and unprecedented chest pain a client whose asthma is treated with inhaled corticosteroids

a client who is reporting pain near the surgical site Explanation: A report of "breakthrough" pain, especially postsurgery, would likely require the nurse to administer a PRN analgesic. A new onset of chest pain would likely require a stat order, while longstanding treatment of hypertension and asthma would likely include standing orders for relevant medications. Reference: Chapter 29: Medications - Page 831-832

A client finished a course of antibiotics for laryngitis but continues to experience persistent hoarseness. Which symptom would cause the nurse to suspect laryngeal cancer? headaches in the morning discomfort when drinking cold liquids a feeling of swelling at the back of the throat weight loss

a feeling of swelling at the back of the throat Explanation: After an initial hoarseness lasting longer than a month, clients with laryngeal cancer will feel a sensation of swelling or a lump in the throat or in the neck. Weight loss often occurs later in the progression of laryngeal cancer due to reduced calorie intake as a result of impaired swallowing and pain. Clients with laryngeal cancer may report burning in the throat when swallowing hot or citrus liquids. Clients with obstructive sleep apnea may experience a morning headache. Reference: Hinkle, J.L., and Cheever, K.H. Brunner & Suddarth's Textbook of Medical-Surgical Nursing, 14th ed. Philadelphia: Lippincott Williams & Wilkins, 2018, Chapter 22: Management of Patients With Upper Respiratory Tract Disorders, p. 572. Chapter 22: Management of Patients With Upper Respiratory Tract Disorders - Page 572

Laboratory testing is ordered for a male client during a clinic visit for routine follow-up assessment of hypertension. When interpreting lab values, the nurse knows: if the result of a very sensitive test is negative, that does not mean the person is disease free. if the lab result is above the 50% distribution, the result is considered elevated. a normal value represents the test results that fall within the bell curve. all lab values are adjusted for gender and weight.

a normal value represents the test results that fall within the bell curve. Explanation: What is termed a normal value for a laboratory test is established statistically from results obtained from a selected sample of people. A normal value represents the test results that fall within the bell curve or the 95% distribution. Some lab values (like hemoglobin) are adjusted for gender, other comorbidities, or age. If the result of a very sensitive test is negative, it tells us the person does not have the disease and the disease has been ruled out or excluded. Reference: Chapter 1: Concepts of Health and Disease - Page 1

A client who had intracavity radiation treatment for cervical cancer 1 month earlier reports small amounts of vaginal bleeding. This finding most likely represents: an expected effect of the radiation therapy. development of a rectovaginal fistula. recurrence of the carcinoma. infection secondary to a change in vaginal flora.

an expected effect of the radiation therapy. Explanation: After intracavity radiation, some vaginal bleeding occurs for 1 to 3 months. Intermittent, painless vaginal bleeding is a classic symptom of cervical cancer, but given the client's history, bleeding in more likely a result of the radiation. The passage of feces through the vagina, not vaginal bleeding, is a sign of rectovaginal fistula. Vaginal infections are indicated by various types of vaginal discharge, not vaginal bleeding. Reference: Hinkle, J.L., and Cheever, K.H. Brunner & Suddarth's Textbook of Medical-Surgical Nursing, 14th ed., Philadelphia: Wolters Kluwer, 2018, Chapter 57: Management of Patients With Female Reproductive Disorders, Internal (Intracavitary) Irradiation, p. 1716. Chapter 57: Management of Patients With Female Reproductive Disorders - Page 1716 Add a Note

For which client would the use of standard precautions alone be appropriate? an incontinent client in a nursing home who has diarrhea a child with chickenpox who is treated in the emergency room a client with TB who needs medications administered a client with diphtheria who needs p.m. care

an incontinent client in a nursing home who has diarrhea Explanation: Standard precautions apply to blood and all body fluids, secretions, and excretions except sweat. Transmission-based precautions are used in addition to standard precautions for clients hospitalized with suspected infection by pathogens that can be transmitted by airborne, droplet, or contact routes, such as is the case in answers A, B, and D. Reference: Chapter 24: Asepsis and Infection Control - Page 614-615

The nurse notes that the client demonstrates generalized pallor and recognizes that this finding may be indicative of local arterial insufficiency. vitiligo. albinism. anemia.

anemia. Explanation: Reference: Hinkle, J.L., and Cheever, K.H. Brunner & Suddarth's Textbook of Medical-Surgical Nursing, 14th ed., Philadelphia: Wolters Kluwer, 2018, Chapter 60: Assessment of Integumentary Function, Table 60-1, p. 1797. Chapter 60: Assessment of Integumentary Function - Page 1797

A client has just returned to the unit following abdominal surgery and is in significant pain. According to the nursing process, how frequently will the nurse perform assessments on this client? once upon arrival and every 2 hours afterward as often as needed twice per shift once upon arrival and 1hour later

as often as needed Explanation: Assessment is an important, recurring nursing activity that continues as long as a need for healthcare exists. During assessment, the nurse methodically obtains data about the client's health, illness, and change in condition. Reference: Hinkle, J.L., and Cheever, K.H. Brunner & Suddarth's Textbook of Medical-Surgical Nursing, 14th ed. Philadelphia: Lippincott Williams & Wilkins, 2018, Chapter 3: Critical Thinking, Ethical Decision Making, and the Nursing Process, p. 37. Chapter 3: Critical Thinking, Ethical Decision Making, and the Nursing Process - Page 37

Seven hours ago, a multigravida woman gave birth to a male infant weighing 4,133 g. She has voided once and calls for a nurse to check because she states that she feels "really wet" now. Upon examination, her perineal pad is saturated. The immediate nursing action is to: call the primary care provider or the nurse-midwife. assess and massage the fundus. increase the flow of an IV. inspect the perineum for lacerations.

assess and massage the fundus. Explanation: This woman is a multigravida who gave birth to a large baby and is at risk for hemorrhage. The other actions are to be done after the initial fundal massage. Reference: Ricci, S. S., Kyle, T., Carman, S. Maternity and Pediatric Nursing, 3rd ed. Philadelphia: Wolters Kluwer Health, 2017, Chapter 16: Nursing Management During the Postpartum Period, p. 562. Chapter 16: Nursing Management During the Postpartum Period - Page 562

A client had a nephrectomy 2 days ago and is now complaining of abdominal pressure and nausea. The first nursing action should be to change the client's position. insert a rectal tube. palpate the abdomen. auscultate bowel sounds.

auscultate bowel sounds. Explanation: If abdominal distention is accompanied by nausea, the nurse must first auscultate bowel sounds. If bowel sounds are absent, the nurse should suspect gastric or small intestine dilation and these findings must be reported to the physician. Palpation should be avoided postoperatively with abdominal distention. If peristalsis is absent, changing positions and inserting a rectal tube won't relieve the client's discomfort.

It is important for the nurse to assist a postsurgical client to sit up and turn the head to one side when vomiting in order to help eliminate inhaled anesthetics. avoid dizziness. avoid aspiration. maximize comfort.

avoid aspiration. Explanation: The nurse helps the client to sit up and turn the head to one side when vomiting in order to avoid aspiration. This does not maximize comfort and does not help to avoid dizziness. Encouraging the client to breathe deeply helps eliminate inhaled anesthetics. Reference: Hinkle, J.L., and Cheever, K.H. Brunner & Suddarth's Textbook of Medical-Surgical Nursing, 14th ed., Philadelphia: Wolters Kluwer, 2018, Chapter 19: Postoperative Nursing Management, p. 458. Chapter 19: Postoperative Nursing Management - Page 458

A nurse notices that a client admitted for exacerbation of chronic obstructive pulmonary disease is short of breath. The client has signed an advance directive indicating that they don't want to be resuscitated. The nurse should get the crash cart. call the physician. check the client's oxygen saturation. not provide any care.

check the client's oxygen saturation. Explanation: The nurse should check the client's oxygen saturation before calling the physician. The fact that the client has signed an advance directive doesn't mean that the nurse shouldn't provide any care. There's no reason for the nurse to get the crash cart at this point.

What is the term for the ability of the cardiac muscle to shorten in response to an electrical impulse? contractility depolarization repolarization diastole

contractility Explanation: Contractility is the ability of the cardiac muscle to shorten in response to an electrical impulse. Depolarization is the electrical activation of a cell caused by the influx of sodium into the cell while potassium exits the cell. Repolarization is the return of the cell to the resting state, caused by reentry of potassium into the cell while sodium exits the cell. Diastole is the period of ventricular relaxation resulting in ventricular filling. Reference: Hinkle, J.L., and Cheever, K.H. Brunner & Suddarth's Textbook of Medical-Surgical Nursing, 14th ed., Philadelphia: Wolters Kluwer, 2018, Chapter 25: Assessment of Cardiovascular Function, p. 672. Chapter 25: Assessment of Cardiovascular Function - Page 672 Add a Note

A client is concerned about the lumps that have developed in her breasts and is fearful of cancer. The client reports variability in the size of the lumps. What could be causing this condition? progesterone cyclical hormonal changes caffeine nicotine

cyclical hormonal changes Explanation: The likely cause is fibrocystic disease, which results from hormonal changes during the menstrual cycle. Reference: Hinkle, J.L., and Cheever, K.H. Brunner & Suddarth's Textbook of Medical-Surgical Nursing, 14th ed., Philadelphia: Wolters Kluwer, 2018, Chapter 58: Assessment and Management of Patients With Breast Disorders, p. 1728. Chapter 58: Assessment and Management of Patients With Breast Disorders - Page 1728

A client undergoes a craniotomy with supratentorial surgery to remove a brain tumor. On the first postoperative day, the nurse notes the absence of a bone flap at the operative site. How should the nurse position the client's head? turned onto the operative side flat elevated no more than 10 degrees elevated 30 degrees

elevated 30 degrees Explanation: After supratentorial surgery, the nurse should elevate the client's head 30 degrees to promote venous outflow through the jugular veins. The nurse would keep the client's head flat after infratentorial, not supratentorial, surgery. However, after supratentorial surgery to remove a chronic subdural hematoma, the neurosurgeon may order the nurse to keep the client's head flat; typically, the client with such a hematoma is older and has a less expandable brain. A client without a bone flap can't be positioned with the head turned onto the operative side because doing so may injure brain tissue. Elevating the head 10 degrees or less wouldn't promote venous outflow through the jugular veins. Remediation:

A benign tumor of the blood vessels is a(n) neuroma. osteoma. chondroma. hemangioma. SUBMIT ANSWER

hemangioma. Explanation: A hemangioma is a benign tumor of the blood vessels. An osteoma is a tumor of the connective tissue. A neuroma is a tumor of the nerve cells. A chondroma is a tumor of the cartilage. Reference: Hinkle, J.L., & Cheever, K.H., Brunner & Suddarth's Textbook of Medical-Surgical Nursing, 14th ed., Philadelphia, Wolters Kluwer, 2018.

A child with type 1 diabetes develops diabetic ketoacidosis and receives a continuous insulin infusion. Which condition represents the greatest risk to this child? hypokalemia hypernatremia hyperphosphatemia hypercalcemia

hypokalemia Explanation: Hypokalemia occurs when insulin administration causes glucose and potassium to move into the cells. Insulin administration doesn't directly affect calcium levels. Hypophosphatemia — not hyperphosphatemia — may occur with insulin administration because phosphorus enters the cells with insulin and potassium. Insulin administration doesn't directly affect sodium levels. Remediation:

A nurse is caring for a client with diabetes mellitus. The client takes insulin 2 times per day. The nurse makes sure the client's meals arrive in coordination with the insulin's effect. The knowledge used by the nurse is: lacking. evaluative. creative. integrated.

integrated. Explanation: This scenario indicates the integration of a nurse's knowledge in the provision of safe client care. Reference: Taylor, C., et al. Fundamentals of Nursing, 8th ed. Philadelphia: Wolters Kluwer Health/Lippincott Williams & Wilkins; 2015,

A nurse is reviewing a client's morning laboratory results and notes a left shift in the band cells. Based on this result, the nurse can interpret that the client may be developing anemia. may be developing an infection. has thrombocytopenia. has leukopenia.

may be developing an infection. Explanation: Less mature granulocytes have a single-lobed, elongated nucleus and are called band cells. Ordinarily, band cells account for only a small percentage of circulating granulocytes, although their percentage can increase greatly under conditions in which neutrophil production increases, such as infection. An increased number of band cells is sometimes called a left shift or shift to the left. Anemia refers to decreased red cell mass. Leukopenia refers to a less-than-normal amount of white blood cells in circulation. Thrombocytopenia refers to a lower-than-normal platelet count. Reference: Hinkle, J.L., and Cheever, K.H. Brunner & Suddarth's Textbook of Medical-Surgical Nursing, 14th ed., Philadelphia: Wolters Kluwer, 2018, Chapter 32: Assessment of Hematologic Function and Treatment Modalities, p. 906. Chapter 32: Assessment of Hematologic Function and Treatment Modalities - Page 906

An incoherent client with a history of hypothyroidism is brought to the emergency department by the rescue squad. Physical and laboratory findings reveal hypothermia, hypoventilation, respiratory acidosis, bradycardia, hypotension, and nonpitting edema of the face and periorbital area. Knowing that these findings suggest severe hypothyroidism, the nurse prepares to take emergency action to prevent the potential complication of: thyroid storm. myxedema coma. Hashimoto's thyroiditis. cretinism.

myxedema coma. Explanation: Severe hypothyroidism may result in myxedema coma, in which a drastic drop in the metabolic rate causes decreased vital signs, hypoventilation (possibly leading to respiratory acidosis), and nonpitting edema. Thyroid storm is an acute complication of hyperthyroidism. Cretinism is a form of hypothyroidism that occurs in infants. Hashimoto's thyroiditis is a common chronic inflammatory disease of the thyroid gland in which autoimmune factors play a prominent role. Reference: Hinkle, J.L., and Cheever, K.H. Brunner & Suddarth's Textbook of Medical-Surgical Nursing, 14th ed., Philadelphia: Wolters Kluwer, 2018, Chapter 52: Assessment and Management of Patients With Endocrine Disorders, Clinical Manifestations, p. 1514. Chapter 52: Assessment and Management of Patients With Endocrine Disorders - Page 1514

The school nurse has several children with hemophilia A. After recess, one hemophilia student comes to the school nurse complaining of pain in the knee from falling on the playground. The nurse notes there is swelling in the knee and pain on palpation. The nurse should: administer some NSAIDs to relieve the pain. wrap the knee in an ace bandage for compression. notify parents to pick up the child and possibly administer Factor VIII. apply some warm compresses to the knee.

notify parents to pick up the child and possibly administer Factor VIII. Explanation: Prevention of trauma is important. ASA and other NSAIDs that affect platelet function should be avoided. Factor VIII replacement therapy administered at home has reduced the typical musculoskeletal damage. Wrapping with a bandage will not prevent damage. Warm compression will extend the bleed. Reference: Chapter 22: Disorders of Hemostasis - Page 556

What is the nursing diagnosis the framework for? outcome statements evaluation summations medical interventions nursing interventions

nursing interventions Explanation: A nursing diagnosis identifies problems that can be solved or prevented by independent nursing actions. The nursing diagnosis provides the framework for the selection of nursing interventions. Regarding the other answers, a medical intervention is different than a nursing intervention, evaluation occurs after interventions have happened, and evaluation reviews the outcomes. Reference: Chapter 4: The Nursing Process in Drug Therapy and Patient Safety - Page 48-49

A client is to receive general anesthesia with sevoflurane. What does the nurse anticipate would be given with the inhaled anesthesia? alfentanil lidocaine oxygen rocuronium

oxygen Explanation: Sevoflurane is an inhalation anesthetic always combined with oxygen to decrease the risk of coughing and laryngospasm. It would not be combined with alfentanil, rocuronium, or lidocaine. Alfentanil and rocuronium are intravenous anesthetics. Lidocaine is a local anesthetic. Reference: Hinkle, J.L., and Cheever, K.H. Brunner & Suddarth's Textbook of Medical-Surgical Nursing, 14th ed., Philadelphia: Wolters Kluwer, 2018, Chapter 18: Intraoperative Nursing Management, Table 18-1, p. 444. Chapter 18: Intraoperative Nursing Management - Page 444

A 70-year-old woman on long-term ibuprofen therapy for osteoarthritis has returned to the clinic for her regular 6-month visit. The client states that in the last couple of months, she has been having increasing periods of abdominal pain. The nurse suspects that this pain may be related to which? peptic ulcer disease or gastritis. constipation. anemia. interstitial nephritis.

peptic ulcer disease or gastritis. Explanation: During long-term ibuprofen therapy, especially in clients older than 60 years, the nurse needs to closely monitor for peptic ulcer disease or gastritis that can lead to gastrointestinal bleeding or even bowel perforation. These events can occur at any time, with or without warning. Ibuprofen may also cause excessive or abnormal bleeding, especially in clients with anemia, but it is not known to cause anemia. Interstitial nephritis is one of the less common renal toxicities associated with ibuprofen as is constipation. Reference: Chapter 16: Anti-Inflammatory, Antiarthritis, and Related Agents - Page 279

A nurse is caring for a client in the postpartum period. When observing the client's condition, the nurse notices that the client tends to speak incoherently. The client's thought process is disoriented, and she frequently indulges in obsessive concerns. The nurse notes that the client has difficulty in relaxing and sleeping. The nurse interprets these findings as suggesting which condition? postpartum panic disorder postpartum psychosis postpartum depression postpartum blues

postpartum psychosis Explanation: The client's signs and symptoms suggest that the client has developed postpartum psychosis. Postpartum psychosis is characterized by clients exhibiting suspicious and incoherent behavior, confusion, irrational statements, and obsessive concerns about the baby's health and welfare. Delusions, specific to the infant, are present. Sudden terror and a sense of impending doom are characteristic of postpartum panic disorders. Postpartum depression is characterized by a client feeling that her life is rapidly tumbling out of control. The client thinks of herself as an incompetent parent. Emotional swings, crying easily—often for no reason—and feelings of restlessness, fatigue, difficulty sleeping, headache, anxiety, loss of appetite, decreased ability to concentrate, irritability, sadness, and anger are common findings are characteristic of postpartum blues. Reference: Ricci, S. S., Kyle, T., Carman, S. Maternity and Pediatric Nursing, 3rd ed. Philadelphia: Wolters Kluwer Health, 2017, Chapter 22: Nursing Management of the Postpartum Woman at Risk, p. 860. Chapter 22: Nursing Management of the Postpartum Woman at Risk - Page 860

A nurse notices that a client's left upper eyelid is drooping. The nurse has observed: proptosis ptolemy ptosis nystagmus

ptosis Explanation: Ptosis is drooping or falling of the upper or lower eyelid. Ptolemy is not a medical condition. Proptosis is the extended or protruded upper eyelid that delays closing or remains partially open. Nystagmus is uncontrolled oscillating movement of the eyeball. Reference: Hinkle, J.L., and Cheever, K.H. Brunner & Suddarth's Textbook of Medical-Surgical Nursing, 14th ed., Philadelphia: Wolters Kluwer, 2018, Chapter 63: Assessment and Management of Patients With Eye and Vision Disorders, p. 1880. Chapter 63: Assessment and Management of Patients With Eye and Vision Disorders - Page 1880

The client with herpes simplex virus (HSV) encephalitis is receiving acyclovir. The nurse monitors blood chemistry test results and urinary output for signs of relapse. signs and symptoms of cardiac insufficiency. signs of improvement in the patient's condition. renal complications related to acyclovir therapy.

renal complications related to acyclovir therapy. Explanation: Monitoring of blood chemistry test results and urinary output will alert the nurse to the presence of renal complications related to acyclovir therapy. To prevent relapse, treatment with acyclovir should continue for up to 3 weeks. Reference: Hinkle, J.L., and Cheever, K.H. Brunner & Suddarth's Textbook of Medical-Surgical Nursing, 14th ed., Philadelphia: Wolters Kluwer, 2018, Chapter 69: Management of Patients With Neurologic Infections, Autoimmune Disorders, and Neuropathies, p. 2070. Chapter 69: Management of Patients With Neurologic Infections, Autoimmune Disorders, and Neuropathies - Page 2070

The nurse is caring for a client immediately after supratentorial intracranial surgery. The nurse performs the appropriate action by placing the patient in the supine position with the head slightly elevated. prone position with the head turned to the unaffected side. Trendelenburg position. dorsal recumbent position.

supine position with the head slightly elevated. Explanation: After surgery, the nurse should place the client in either a supine position with the head slightly elevated or a side-lying position on the unaffected side. The dorsal recumbent, Trendelenburg, and prone positions can increase intracranial pressure. Reference: Hinkle, J.L., and Cheever, K.H. Brunner & Suddarth's Textbook of Medical-Surgical Nursing, 14th ed., Philadelphia: Wolters Kluwer, 2018, Chapter 66: Management of Patients With Neurologic Dysfunction, p. 1988. Chapter 66: Management of Patients With Neurologic Dysfunction - Page 1988

After receiving the shift report, a registered nurse in the cardiac step-down unit must prioritize the client care assignment. The nurse has an ancillary staff member available to help care for the clients. Which of these clients should the registered nurse assess first? the client admitted during the previous shift with new-onset controlled atrial fibrillation, who has a call light on the coronary bypass client asking for pain medication for "11 of 10" pain in the donor site the client with heart failure who is having some difficulty breathing the anxious client who was diagnosed with an acute myocardial infarction (MI) 2 days ago, and was transferred from the coronary care unit today

the client with heart failure who is having some difficulty breathing Explanation: The registered nurse should care for the client with heart failure who is experiencing difficulty breathing. Breathing takes precedence over the other client needs. Although anxiety can be detrimental to a client with myocardial infarction, anxiety does not take precedence over another client's breathing difficulty. The ancillary staff member can answer the call light of the client admitted with controlled atrial fibrillation. The coronary bypass client in pain needs an analgesic, but that does not take priority over a client with difficulty breathing.

A fundal massage is sometimes performed on a postpartum woman. The nurse would perform this procedure to address which condition? uterine prolapse uterine contraction uterine subinvolution uterine atony

uterine atony Explanation: Fundal massage is performed for uterine atony, which is failure of the uterus to contract and retract after birth. The nurse would place the gloved dominant hand on the fundus and the gloved nondominant hand on the area just above the symphysis pubis. Using a circular motion, the nurse massages the fundus with the dominant hand. Then the nurse checks for firmness and, if firm, applies gentle downward pressure to express clots that may have accumulated. Finally, the nurse assists the woman with perineal care and applying a new perineal pad. Reference: Ricci, S. S., Kyle, T., Carman, S. Maternity and Pediatric Nursing, 3rd ed. Philadelphia: Wolters Kluwer Health, 2017, Chapter 22: Nursing Management of the Postpartum Woman at Risk, p. 845. Chapter 22: Nursing Management of the Postpartum Woman at Risk - Page 845

The nurse has just completed an assessment on a client admitted with Guillain-Barré syndrome. The nurse determines that a priority of care will be: ventilatory assessment and support. feeding the client. administration of antibiotics. emotional support.

ventilatory assessment and support. Explanation: Guillain-Barré syndrome usually is a medical emergency. There may be a rapid development of ventilatory failure and autonomic disturbances that threaten circulatory function. Assessment of the airway is the top priority. Treatment includes support of vital functions and prevention of complications such as skin breakdown and thrombophlebitis. Reference: Chapter 15: Disorders of Motor Function - Page 393


Kaugnay na mga set ng pag-aaral

C724: UNIT 6: Systems Development and Decision Making - UNIT TEST

View Set

Chapter exam premiums and proceeds

View Set

Patho Ch 36 Dynamic Study Module Chronic Musculoskeletal Disorders

View Set